Jump to content

MRCP


veterinarian

Recommended Posts

MRCP May 2003

1) A 68 year old male diagnosed with nephritic syndrome receives steroid therapy without benefit. His investigations show an albumin of 20 g/L (37 - 49), Total cholesterol of 12 mmol/l, dipstick urinanalysis reveals +++ protein and a renal biopsy shows focal segmental glomerulosclerosis. Which one of the following is most likely to preserve renal function?

1 ) dietary salt restriction

2 ) low dietary protein intake

3 ) ramipril

4 ) simvastatin

5 ) warfarin

2) A 80 year-old man presents with deteriorating lumbar and bilateral hip pains. He has recently been seen by his GP with symptoms of prostatism.

Investigations revealed:

full blood count normal

Corrected calcium 2.3 mmol/L (2.2 – 2.6)

ESR 22 mm/1st hr (1-10)

Alkaline phosphatase 985 iu/L (50 – 100)

gamma-GT 33 iu/L (10-50)

Prostate specific antigen 6.6 pg/L (0-6)

What is the most likely cause of this man's pain?

1 ) Osteomalacia

2 ) Multiple myeloma

3 ) Paget's disease of the pelvis

4 ) Polymyalgia rheumatica

5 ) Prostatic carcinoma with metastases

3) A 60 year old male is brought to casualty in the early hours of the morning after being found unconscious in the street. On examination, he was drowsy but localised to painful stimuli. There was no evidence of head injury or meningism. Investigations revealed:

sodium 134 mmol/L (137-144)

potassium 4.0 mmol/L (3.5-4.9)

urea 4.0 mmol/L (2.5-7.5)

creatinine 80 micromol/L(60-110)

glucose 4.5 mmol/L (3.0-6.0)

chloride 100 mmol/L (95 - 107)

bicarbonate 25 mmol/L (20 - 28)

plasma osmolality 385 mosmol/Kg (278 - 305)

What is the most likely explanation for his presentation?

1 ) diazepam poisoning

2 ) ethanol poisoning

3 ) methanol poisoning

4 ) phenobarbitone poisoning

5 ) Phenytoin poisoning

4) A 29-year-old male presents with symptoms of severe gastro-oesophageal reflux. Which one of the following is most useful in assessing the role of surgery?

1 ) cardiac sphincter manometry

2 ) gastric emptying study

3 ) intragastric pH monitoring off therapy

4 ) oesophageal motility study

5 ) oesophageal pH monitoring on therapy

5) A 70 year-old female is diagnosed with anaplastic thyroid cancer. What is the most likely consequence of this cancer?

1 ) Brain metastases

2 ) Hypercalcaemia from bony metastases

3 ) Liver metastases

4 ) Lung metastases

5 ) Upper airways obstruction

6) A 65 year old female who has a history of longstanding psoriasis and heavy alcohol intake, presents with a severe exacerbation of psoriasis. She was admitted and received topical therapy and over the month of her admission, her gamma-GT concentration had fallen from 400 U/L to 150 U/L (4 - 35). Six weeks after discharge she was seen in outpatients where her psoriasis remained under control, but she complained of generalized hair loss. What is the most likely cause for her hair loss?

1 ) Alopecia areata

2 ) iron deficiency

3 ) telogen effluvium

4 ) thiamine deficiency

5 ) trichotillomania

7) A 46-year-old male was seen for an insurance medical examination. He was entirely asymptomatic, but his serum urate concentration was noted to be 0.5 mmol/L (0.23 - 0.46). What is the most appropriate management for this patient?

1 ) Allopurinol

2 ) Colchicine

3 ) Ibuprofen

4 ) Lifestyle intervention

5 ) Sulphinpyrazone

8) An 80 year old man with a 5 year history of diet controlled type 2 diabetes mellitus presents with a one month history of cough and weight loss. He was a non-smoker and had difficulty expectorating. Investigation revealed a HbA1c of 7% but his chest X-ray showed a cavitating left apical shadow. Which of the following investigations would be most useful in establishing the cause of this lesion?

1 ) bronchoscopy

2 ) CT scan of the chest

3 ) Gastric aspirate for acid-fast bacilli

4 ) Percutaneous lung biopsy

5 ) Sputum for acid-fast bacilli

9) An 84-year-old woman presented with tiredness. On examination, she was anaemic but had no palpable splenomegaly. Investigations revealed a haemoglobin of 9.7 g/dL (11.5 - 16.5). She was commenced on oral iron therapy for one month and her haemoglobin remained unchanged. Further investigations revealed:

MCV 102 fL (80 - 96)

blood film marked anisopoikilocytosis

serum ferritin 70 ug/L(15 - 300)

Vitamin B12 280 ng/L (160 - 760)

red cell folate 230 ug/L (160 - 640)

serum urea 9.1 mmol/L (2.5-7.5)

serum creatinine 150 umol/L (60 - 110)

What is the most likely diagnosis?

1 ) aplastic anaemia

2 ) anaemia due to renal disease

3 ) hypothyroidism

4 ) iron deficiency anaemia

5 ) sideroblastic anaemia

10) A 17 year-old girl presents with vomiting and her investigations show:

sodium 120 mmol/L (137 - 144)

potassium 3.0 mmol/L (3.5-4.9)

urea 2.2 mmol/L (2.5 - 7.5)

urine sodium 2 mmol/L

urine osmolality 700 mosmol/kg (350 - 1000)

What is the most likely diagnosis?

1 ) Addison's disease

2 ) bulimia nervosa

3 ) diuretic abuse

4 ) syndrome of inappropriate antidiuretic hormone secretion

5 ) water intoxication

11) A 60-year-old woman diagnosed with giant cell arteritis was commenced on high dose prednisolone therapy. What is the most appropriate treatment for the prevention of steroid-induced osteoporosis?

1 ) Bisphosphonate therapy

2 ) Calcium and vitamin D

3 ) Hormone replacement therapy

4 ) Raloxifene

5 ) Salmon Calcitonin

12) A clinical trial assessing a new lipid lowering therapy for stroke allocates 1000 patients to active treatment and another 1000 patients to placebo. Results demonstrate that number needed to treat (NNT) is 20 for the prevention of the primary end-point. Which of the following best describes the results?

1 ) 20 patients in the treatment group were protected from stroke.

2 ) 20 extra patients in the placebo group had a stroke

3 ) For 1000 patients treated with active therapy, there would be 20 fewer strokes

4 ) For 1000 patients treated with active therapy, there would be 50 fewer strokes.

5 ) For every 1000 patients treated with active therapy there would be 100 fewer strokes

13) Which of the following statements concerning iron metabolism is correct?

1 ) Approximately 0.1% of body iron circulates in the plasma

2 ) Approximately 90% of dietary iron is absorbed in the intestine

3 ) The main route of excretion is the liver

4 ) The serum ferritin concentration is reduced characteristically following surgery

5 ) The transferrin content of intestinal mucosal cells is high when body iron stores are high

14) An 80 year-old woman with type 2 diabetes mellitus is referred with weakness. She had been taking bendrofluazide, digoxin and tolbutamide. On examination she had a temperature of 37.8°C, a pulse of 98 bpm in atrial fibrillation, and a blood pressure of 118/72 mmHg.

Investigations show:

Sodium 121 mmol/L (137 - 144)

Potassium 3.3 mmol/L (3.5 - 4.9)

Urea 4.8 mmol/L (2.5-7.5)

Creatinine 83 micromol/L (60 - 110)

Glucose 15.2 mmol/L (3.0 - 6.0)

chest X-ray normal

What is the most likely cause for the hyponatraemia?

1 ) Addison's disease

2 ) bendrofluazide (bendroflumethiazide)

3 ) hyperglycaemia

4 ) syndrome of inappropriate secretion of antidiuretic hormone

5 ) tolbutamide

15) Which of the following is characteristically inherited in an autosomal recessive manner?

1 ) Achondroplasia

2 ) Adult polycystic kidney disease

3 ) C1 esterase deficiency

4 ) Familial hypercholesterolaemia

5 ) Friedreich's ataxia

16) A 40 year old male presents with a long history of productive cough and breathlessness. He had complained of halitosis and exacerbations of productive sputum, chest pain and haemoptysis. Examination revealed bilateral inspiratory crackles. Which of the following treatments is likely to decrease the frequency of his exacerbations?

1 ) cyclical antibiotic therapy

2 ) inhaled corticosteroids

3 ) nebulised bronchodilators

4 ) postural drainage

5 ) surgical resection

17) A 35 year-old woman with type 1 diabetes mellitus presents for annual assessment. Which one of the following features on fundoscopy would require urgent referral to an ophthalmologist?

1 ) asteroid bodies

2 ) hard exudates in the macular region

3 ) intraretinal microvascular abnormalities

4 ) scattered microaneurysms

5 ) soft exudates

18) A 70 year old male presents with haemoptysis. Bronchoscopy reveals a tumour in the proximal right main bronchus. Which of the following is a contraindication to radical radiotherapy?

1 ) Adenocarcinoma

2 ) FEV1 of 25% predicted

3 ) involvement of the pulmonary artery

4 ) ischaemic heart disease

5 ) superior vena caval obstruction

19) A 60 year-old male with diet controlled type 2 diabetes mellitus is commenced on metformin due to deteriorating glycaemic control. Which of the following is true regarding metformin?

1 ) It often causes hypoglycaemia

2 ) It is safe in patients with renal impairment

3 ) It may cause metabolic alkalosis

4 ) It is contra-indicated in patients suffering a myocardial infarction

5 ) It does not require any functioning pancreatic islet cells for its action

20) A 30 year old male PE teacher with a 15 year history of type I diabetes mellitus is referred by his GP with microalbuminuria. He takes humulin M3 (30/70 mix) twice daily and his blood pressure is noted to be 132/78 mmHg with a BMI of 28. His HbA1c is 6.7%. What is the most appropriate treatment for this patient?

1 ) ACE inhibitor therapy

2 ) Add metformin

3 ) Avoid severe exercise

4 ) Switch to four times daily insulin

5 ) Switch to four times daily insulin

21) These are the blood gas results obtained in a 20 year old female admitted to hospital.

hydrogen ion concentration 35 nmol/L (35 – 45)

pH 7.45 (7.35-7.45)

pC02 6.8 kPa (4.6 – 5.9)

bicarbonate 32 mmol/L (22 - 26)

Which one of the following is a recognised cause of this acid-base disorder?

1 ) Amitriptyline overdose

2 ) Cushing's syndrome

3 ) Hepatic failure

4 ) Pregnancy

5 ) Salicylate poisoning

22) A 34-year-old man presented for an insurance medical. He was symptom free, but clinical examination suggested a small ventricular septal defect. Which one of the following findings was most likely to have been present?

1 ) An early diastolic murmur

2 ) A short systolic murmur at the left sternal edge

3 ) A systolic murmur maximal at the apex

4 ) A systolic murmur with thrill at the left sternal edge

5 ) Fixed splitting of the second heart sound

23) A 15 year old girl presents with fever, malaise and sore throat. Examination revals a temperature of 38.3°C with cervical lymphadenopathy. Her results show:

Haemoglobin 12.8 g/dl(11.5-16)

White cell count 9.8x109/l (4-11)

Neutrophils 3x109/l

Lymphocytes 4.5x109/l

Blood film reveals atypical mononuclear cells

What is the most likely diagnosis?

1 ) Acute Lymphoblastic Leukaemia

2 ) Brucellosis

3 ) Epstein-Barr viral infection

4 ) Hodgkin's disease

5 ) Sarcoidosis

24) A 65 year old female presents with heart failure. Her echocardiogram shows a restrictive cardiomyopathy but with structurally normal valves. Which one of the following is the most likely cause?

1 ) amyloidosis

2 ) coxsackie infection

3 ) Down’s syndrome

4 ) Marfan's syndrome

5 ) Turner's syndrome

25) A 38 year old male presents with episodic wheeze and non-productive cough which occurs particularly at night. He has been employed in the plastics industry. Which of the following suggests a diagnosis of occupational lung disease?

1 ) Absent family history of asthma

2 ) Commencement of symptoms on his first day in this employment

3 ) elevated serum IgE concentration

4 ) Improved symptomatology when on holiday

5 ) Increased bronchial reactivity

26) A 40 year old ex-footballer presents requesting treatment for alcoholism and is prescribed disulfiram. What is the mode of action of Disulfiram?

1 ) Decreases severity of alcohol withdrawal

2 ) Helps alcoholics to drink safely

3 ) Inhibits acetaldehyde dehydrogenase activity

4 ) Inhibits alcohol dehydrogenase activity

5 ) Reduces the desire for alcohol

27) A 52 year-old male presents with general deterioration. He drinks approximately 25 units of alcohol each week and is a smoker of 5 cigarettes daily. Examination reveals that he is jaundiced, has numerous spider naevi on his chest and he has a temperature of 37.2C. Abdominal examination reveals hepato-splenomegaly.

Investigations reveal:

Bilirubin 200 micromol/L (1-22)

Alkaline phosphatase 550 iu/l (45 - 105)

AST 258 iu/l (1 - 31)

Albumin 25 g/L (37 - 49)

hepatitis B virus surface antigen positive

hepatitis B virus e antigen negative

hepatitis B virus DNA undetectable

What is the most likely diagnosis?

1 ) Alcoholic liver disease

2 ) Autoimmune chronic active hepatitis

3 ) Carcinoma of the pancreas

4 ) Chronic hepatitis B infection

5 ) Chronic hepatitis D (delta) infection

28) A 72-year-old woman presented with acute severe chest pain with an ECG revealing ST segment elevation in leads II, III and aVF. She was treated with thrombolysis but two days later became acutely unwell. Examination revealed a loud systolic murmur at the apex which radiated into the axilla with associated pulmonary oedema. What is the most likely diagnosis?

1 ) Acute left ventricular failure

2 ) Cardiogenic shock

3 ) pericarditis

4 ) Ruptured papillary muscle

5 ) Ventricular septal defect

29) A 60 year old male diabetic presents to clinic for advice on prevention of a further heart attack after having sustained a myocardial infarction five years previously. He takes metformin 500 mg tds, bendrofluazide 2.5 mg daily and asprin 150 mg daily. His body mass index was 33.5 kg/m2, with a pulse of 82 beats per minute regular and a blood pressure of 152/92 mmHg. His cholesterol concentration is 3.3 mmol/l (< 5.5). What is the most appropriate strategy for this patient?

1 ) 24 hour ambulatory ECG

2 ) Atorvastatin

3 ) Increase aspirin from 150 mg to 300 mg daily

4 ) Orlistat

5 ) Ramipril

30) A 55 year old man who has received haemodialysis for many years presents with deteriorating discomfort in both shoulders. Past medical history included bilateral carpal tunnel decompression. His Investigations reveal:

haemoglobin 10 g/dl

ESR 30 mm/1st hr (1-10)

C-reactive protein 12mg/L (1-10)

Urate 0.58 mmol/L (less than 0.45)

What is the most likely diagnosis?

1 ) b2 microglobulin amyloidosis

2 ) Gout

3 ) Pseudogout

4 ) Polymyalgia rheumatica

5 ) Osteoarthritis

31) A 58-year-old man presents with a month history of breathlessness. He was a non-smoker. On examination, his temperature was 36.7°C, with a respiratory rate of 20 breaths per minute and normal breath sounds to auscultation and a pulse of 92 bpm. Arterial blood gases on air showed:

pH 7.51 (7.36 - 7.44)

pO2 8.4 kPa (11.3 - 12.6)

pCO2 4.0 kPa (4.7 - 6.0)

What is the most likely diagnosis?

1 ) atypical pneumonia

2 ) fibrosing alveolitis

3 ) hysterical hyperventilation

4 ) inhaled foreign body

5 ) pulmonary thromboembolism

32) A 24-year-old man presents to the Accident & Emergency department and complains of shortness of breath. Before his Chest X-ray is taken he tells the casualty officer that he is known to have an 'azygous lobe'.

What region of the Chest X-ray would expect to see an 'azygous lobe'?

1 ) left lower zone

2 ) left mid zone

3 ) left upper zone

4 ) right lower zone

5 ) right upper zone

33) A 72 year old man presents with an acutely painful right knee. On examination, he had a temperature of 37°C with a hot, swollen right knee. Of releavance amongst his investigations, was his white cell count which was 12.6 x109/l and a knee X-ray revealed reduced joint space and calcification of the articular cartilage. Culture of aspirated fluid revealed no growth. What is the most likely diagnosis?

1 ) gout

2 ) Psoriatic monoarthropathy

3 ) Pseudo-gout

4 ) rheumatoid arthritis

5 ) septic arthritis

34) An 80 year-old male presents with a brief history of weakness and giddiness, following an episode of diarrhoea. He has been taking bendrofluazide for the last 3 years. On examination, his pulse is 100 beats per minute with a blood pressure of 130/80 mmHg (lying) and 100/70 mmHg (standing). Investigations reveal:

Sodium 120 mmol/L

Potassium 5.5 mmol/L

Urea 13 mmol/L

Creatinine 130 umol/L

random plasma glucose 13 mmol/L

What is the most likely cause of the hyponatraemia?

1 ) Bendrofluazide

2 ) Diarrhoea

3 ) Hyperglycaemia

4 ) Hypoadrenalism

5 ) Inappropriate secretion of antidiuretic hormone

35) A 62 year old man seeks an opinion due to a tremor mostly affecting the right hand but also latterly the left hand which has gradually deteriorated over 5 years. Past medical history includes asthma for which he takes inhaled salbutamol, Hypertension for which he takes bendrofluazide and lisinopril and depression for which he takes amitriptyline. He smoked 10 cigarettes daily and drank approximately 15 units of alcohol weekly, noting an improvement in his tremor following alcohol. Examination revealed some mild titubation and a postural tremor in both arms with no worsening during finger-nose testing.

What is the most likely diagnosis?

1 ) Benign essential tremor

2 ) Hyperthyroidism

3 ) Parkinson's disease

4 ) Physiological tremor

5 ) Salbutamol-induced tremor

36) A 20 year old male presents with extensive, coalescing, hypopigmented, slightly scaly lesions on his back and chest. The rash had been present for 2 years and had gradually become more extensive. He had otherwise been in good health. The lesions were not symptomatic but he was concerned about their appearance.

What is the most appropriate treatment for his condition?

1 ) ketoconazole cream

2 ) nystatin cream

3 ) oral itraconazole

4 ) oral terbinafine

5 ) terbinafine cream

37) A 40 year old female with mitral stenosis consults for advice regarding operative procedures. In which of the following circumstances would antibiotic prophylaxis of infective endocarditis be required?

1 ) Cardiac catheterisation

2 ) Caesarian section

3 ) Dental scaling

4 ) Removal of a lipoma

5 ) Termination of pregnancy

38)A 24 year-old female attends clinic complaining of numerous depigmented areas on the arms and legs.

Which of the following diseases is most likely to accompany this skin condition?

1 ) Addison's disease

2 ) Tuberous sclerosis

3 ) Υποπαραθυρεοειδισμός

4 ) Pernicious anaemia

5 ) Systemic lupus erythematosus

39) A 19-year-old man with glucose-6-phosphate dehydrogenase deficiency wishes to travel to Africa. Which one of the following should he be advised to avoid?

1 ) primaquine

2 ) loperamide

3 ) mefloquine

4 ) ibuprofen

5 ) yellow fever vaccine

40) A 70 year-old woman presents with acute back pain followed by weakness of dorsiflexion of her left foot.Where would you expect the associated sensory loss?

1 ) Anterior thigh

2 ) Dorsum of foot

3 ) Perineum

4 ) Posterior calf

5 ) Sole of foot

41) A 16 year old female presents with a six month history of excessive weight gain and weakness. On examination she had central obesity with abdominal striae, a blood pressure of 178/96 mmHg and proximal muscle weakness. Urinalysis showed glucose ++. What is the most appropriate initial investigation for this patient?

1 ) 9am plasma cortisol concentration

2 ) 24 hour urinary free cortisol concentration

3 ) ACTH concentration

4 ) a 1mg overnight dexamethasone suppression test

5 ) a short synacthen test

42) A 40 year old man presents with acute weakness and palpitations. Investigations reveal:

Sodium 143 mmol/L (137 - 144)

Potassium 8.0 mmol/L (3.5-4.9)

Urea 35 mmol/L (2.5 - 7.5)

Creatinine 450 umol/L (60 - 110)

Bicarbonate 5 mmol/L (20 - 28)

What is the best immediate therapy?

1 ) intravenous calcium gluconate

2 ) intravenous dextrose and insulin

3 ) intravenous sodium bicarbonate

4 ) nebulised salbutamol

5 ) rectal calcium resonium

43) A 50-year-old male presented with acute respiratory failure during an episode of acute pancreatitis and was thought to have developed adult respiratory distress syndrome (ARDS). Which of the following would support a diagnosis of ARDS?

1 ) High pulmonary capillary wedge pressure

2 ) High protein pulmonary oedema

3 ) Hypercapnia

4 ) Increased lung compliance

5 ) Normal chest X-ray

44) A 55 year-old woman presents with lethargy, diarrhoea together with joint pains and intermittent fever. These symptoms have developed over the six months during which time she has lost 6 kg in weight. Supraclavicular lymphadenopathy is noted.

What is the most likely diagnosis?

1 ) bacillary dysentery

2 ) campylobacter infection

3 ) Coeliac disease

4 ) giardiasis

5 ) Whipple's disease

Treatment is Cotimoxazole. Bacillarary dysentry and c.jejuni infection is characterised by bloody diarrhoea and is not chronic. Coeliac disease and Giardiasis has no lymph involvement.

45) A 65-year-old woman presented with a malabsorption syndrome. She had a past history of radiotherapy for cervical cancer. Small intestine biopsy reveals - villous atrophy and crypt hypertrophy, chronic inflammatory cell infiltrate of the lamina propria together with increase in intra-epithelial lymphocytes. What is the most likely diagnosis?

1 ) Bacterial overgrowth

2 ) Coeliac disease

3 ) Crohn's disease

4 ) Mesenteric ischaemia

5 ) Radiation enteropathy

46) A 78 year old man presents with an acute onset of severe pain and swelling of the left wrist which had developed since she had a chest infection two weeks previously. On examination, he had a temperature of 38 °C and the left wrist was red, swollen and painful. What is the most appropriate initial investigation?

1 ) erythrocyte sedimentation rate

2 ) Full blood count

3 ) joint aspiration

4 ) serum urate concentration

5 ) X-ray of the joint

47) A 30 year old man is admitted three hours after taking an overdose of amitriptyline and diazepam. On examination he was drowsy with a Glasgow Coma Scale of 8, he had a pulse of 140 beats per minute, a blood pressure of 114/88 mmHg and dilated pupils. His oxygen saturation was 90% on room air. What is the most appropriate initial action for this patient?

1 ) activated charcoal

2 ) CT head scan

3 ) ECG

4 ) IV atenolol

5 ) IV flumazenil

48) A 75 year old female presents with an acute infective exacerbation of her long standing Chronic Obstructive Airways Disease. Blood gas analysis whilst she was receiving Oxygen shows:

pH 7.14

P02 18 kPa (11.3-12.5)

PCO2 10.5 kPa (4.7-6.0)

What is the most appropriate immediate management for this patient?

1 ) CPAP

2 ) Doxapram infusion

3 ) Invasive Ventilation

4 ) nebulised salbutamol with ipratropium

5 ) reduce inspired oxygen concentration

49) A 52 year-old male is admitted with haematemesis and melaena. Examination reveals that he is icteric, confused with a flapping tremor, has signs of chronic liver disease, a pulse rate of 110 bpm and blood pressure of 100/70 mmHg. Abdominal examination reveals ascites. An urgent endoscopy reveals small oesophageal varices, without evidence of bleeding but an oozing portal hypertensive gastropathy. Which of the following measures would be the most appropriate treatment for this patient?

1 ) endoscopic banding

2 ) endoscopic injection of adrenaline

3 ) endoscopic injection of ethanolamine

4 ) oral propranolol

5 ) intravenous vitamin K

50) A 70-year-old woman complained of a rash that had developed over a month. She had otherwise been fit and well. On examination, there were numerous tense, fluid filled blisters over the trunk and limbs, but no mucosal involvement was evident.What is the most likely diagnosis?

1 ) Dermatitis herpetiformis

2 ) Erythema multiforme

3 ) Herpes simplex

4 ) Pemphigoid

5 ) Pemphigus vulgaris

51) Which one of the following conditions is DNA analysis the most useful diagnostic test?

1 ) Adult polycystic kidney disease

2 ) Down's syndrome

3 ) Huntington's chorea

4 ) Hypertrophic Obstructive Cardiomyopathy

5 ) Klinefelter's syndrome

52) A 45 year-old male with type 2 diabetes presented to the clinic as his wife complained that he snored excessively. Which of the following would suggest a diagnosis of obstructive sleep apnoea?

1 ) Daytime sleepiness

2 ) Nasal polyps

3 ) Nocturnal cough

4 ) Poor memory

5 ) Stridor

53) A 60 year old woman presents with raised, erythematous lesions on the limbs and blistering in the mouth and eyes. She had been taking a number of drugs prescribed by her GP. Which may be responsible for her presentation?

1 ) Nifedipine

2 ) paracetamol

3 ) Paroxetine

4 ) Prednisolone

5 ) sulphasalazine

54) An 80-year-old female presents with confusion associated with a chest infection. She received standard treatment, and four days later she developed green, then bloody diarrhoea. Which of the following organisms is most likely to be responsible for her diarrhoea?

1 ) Campylobacter jejuni

2 ) Clostridium difficile

3 ) Escherichia coli 0157

4 ) Methicillin-resistant Staphylococcus aureus

5 ) Vancomycin-resistant enterococcus

55) A 60 year old woman presented with a small right pupil, right ptosis and impaired sweating over the ipsilateral forehead. Sweating on the rest of the face was unaffected. Where is the most likely site of this lesion?

1 ) cervical spinal cord

2 ) common carotid artery

3 ) hypothalamus

4 ) internal carotid artery

5 ) lateral medulla

56) A 65-year-old man was investigated for weight loss and dyspepsia. Endoscopic examination revealed an ulcerated lesion in the stomach and biopsy revealed the presence of a low grade mucosa-associated lymphoma with Helicobacter pylori. Further investigation with CT of chest and abdomen were normal as were bone marrow aspirate and trephine. What is the best treatment option for this patient?

1 ) Eradication therapy for Helicobacter pylori

2 ) IV chemotherapy

3 ) Oral chlorambucil

4 ) Partial gastric resection

5 ) Radiotherapy

57) A 58 year old male presents with acute dyspnoea following a convulsion. On examination his blood pressure was 240/120 mmHg and fundal examination reveals pailloedema with haemorrhages and cotton wool spots. His urea, electrolytes and creatinine are normal but chest X-ray reveals pulmonary oedema and cardiomegaly. Which one of the following is the most appropriate immediate treatment?

1 ) atenolol 50 mg orally

2 ) intravenous labetalol

3 ) intravenous sodium nitroprusside

4 ) nifedipine 5 mg sublingually

5 ) nifedipine LA 30 mg orally

58) A 72 year old man is admitted with fast atrial fibrillation but is receiving treatment with digoxin. An inadequate dose is suspected. A sample of blood is drawn six hours after the last dose of digoxin and a plasma concentration is requested. Which of the following factors explains the six hour wait before measuring the digoxin concetration?

1 ) enterohepatic circulation

2 ) the rate of absorption

3 ) the rate of clearance

4 ) the rate of distribution

5 ) the rate of elimination

59) A 47-year-old man presents with confusion and drowsiness. A diagnosis of hepatic encephalopathy is suspected and treatment with lactulose is begun. Which of the following concerning lactulose is true?

1 ) Absorbed from the gut

2 ) Causes hypermagnesaemia

3 ) Contraindicated in diabetes mellitus

4 ) Inhibits proliferation of ammonia-forming organisms in the gut

5 ) Reduces absorption of spironolactone

60) An 81 -year-old man admitted with a stroke becomes increasingly drowsy after receiving nasogastric feeding for five days. Which biochemical abnormality is the most likely cause of his drowsiness?

1 ) hyperglycaemia

2 ) hypermagnesaemia

3 ) hypernatraemia

4 ) hypocalcaemia

5 ) hypophosphataemia

61) A 70 year old woman is referred with hypertension. Despite lifestyle advice, her GP notes blood pressure recordings averaging 170/100 mmHg. She was commenced on an antihypertensive but one month later complains of symptoms suggestive of postural hypotension. Which one of the following is most likely to be responsible for this side effect?

1 ) amlodipine

2 ) atenolol

3 ) bendrofluazide

4 ) doxazosin

5 ) lisinopril

62) A 70 year-old female presents with a six month history of frontal headaches and weight loss. On examination a bitemporal hemianopia was noted. Which of the following suggest the diagnosis of a pituitary tumour?

1 ) 9am cortisol concentration of 350 nmol/L (200 - 700)

2 ) LH concentration of 44 uL (>30)

3 ) Prolactin concentration of 580 mU/L (50-550)

4 ) Random growth hormone concentration 1.2 mU/L (< 1)

5 ) TSH concentration of 3.8 mU/L (0.5 - 4.5)

63) A 52-year-old man with a history of diabetes mellitus presented with hepatomegaly.

Investigations revealed:

Albumin 30 g/L (35-45)

Total bilirubin 22 umol/L (10-22)

Alkaline Phosphatase 134 iU/L (50-110)

ALT 90 iU/L (10 – 40)

gamma-glutamyi transferase 125 iU/L (10-50)

Ferritin 1450 microg/L (15-400)

Which of the following features would be most suggestive of a diagnosis of haemochromatosis?

1 ) Chondrocalcinosis

2 ) Gynaecomastia

3 ) Migratory polyarthritisD myxoedema

4 ) Myxoedema

5 ) Rash

64) A 60-year-old man takes atenolol for hypertension. Which of the following side effects is he most likely to be aware of two hours after taking atenolol?

1 ) Fatigue

2 ) Hesitancy of micturition

3 ) Nausea

4 ) Orthostatic hypotension

5 ) Somnolence

65) A 55 year old male presented six hours after taking an overdose of Lithium tablets which had been prescribed for a bipolar affective disorder.

On examination he was tremulous, had suffered a convulsion and had a Glasgow coma scale of 12/15. His serum lithium concentration was 5.0 mmol/L (0.5 - 1.0)

What is the most appropriate management of this patient?

1 ) Activated charcoal

2 ) Forced alkaline diuresis

3 ) Furosemide 100 mg intravenously twice daily

4 ) Haemodialysis

5 ) Measure lithium concentration in 2 hours

66) A 60 year old man presents with a 5 day history of lower abdominal pain and diarrhoea. He has a history of chronic obstructive airways disease and has had numerous acute infective exacerbations over the last 3 months.

On examination he was dehydrated, with a temperature of 38.6 °C, a blood pressure of 102/72 mmHg and has a distended, tender abdomen. Which of the following is the most appropriate investigation for this patient?

1 ) Chest X-ray

2 ) Plain abdominal X-ray

3 ) Sigmoidoscopy and biopsy

4 ) Stool microscopy

5 ) Ultrasound scan of the abdomen

67) An overweight, 60-year-old female with an 8 year history of type 2 diabetes mellitus presents with deteriorating glycaemic control. She takes gliclazide 160 mg twice daily. Investigations reveal:

Sodium and potassium normal

serum urea 10 mmol/L (2.5 - 7.5)

serum creatinine 160 &micro;mol/L (60 - 110)

serum alanine transaminase 31 U/L (5 - 35)

serum aspartate transferase 30 U/L (1 - 31)

HbA1C 9.0% (3.8 - 6.4)

Which of the following would be most appropriate additional therapy for improved glycaemic control?

1 ) Acarbose

2 ) guar gum

3 ) metformin

4 ) repaglinide

5 ) rosiglitazone

68) A 60-year-old male who was previously fit and well presented with a six week history of blurring of vision. His investigation revealed a fasting plasma glucose of 12.9 mmol/L (3.0 - 6.0). What is the most likely cause of his blurred vision?

1 ) Cataract

2 ) Maculopathy

3 ) Osmotic changes in the lens

4 ) Proliferative diabetic retinopathy

5 ) Retinal vein thrombosis

69) A 58-year-old female presented with unsteadiness and ataxia and gave a recent history of nausea and epigastric pain for which she had been prescribed an antacid and cimetidine. She was an epileptic and had been well controlled with phenytoin for eight years. She had been also been prescribed amitriptyline for depression, was receiving post-menopausal hormone replacement therapy and was self-medicating with St John's wort. Which of the following drugs is most likely to be responsible for her presentation?

1 ) Amitriptyline

2 ) Antacid

3 ) Cimetidine

4 ) estradiol

5 ) St John's wort

70) A 70 year old feamle presents for investigation of fatigue and weight loss. Investigations reveal:

haemoglobin 9.0 g/dL (11.5 - 16.5)

white cell count 2.0 x l09/L (4-11)

platelet count 250 x 109/L (150 - 400)

total protein 74 g/L (61 - 76)

albumin 28 g/L (37 - 49)

urea 16 mmol/l (3-7)

creatinine 250 micromol/L (60 - 110)

plasma glucose 6.5 mmol/L (3.0 - 6.0)

urine dipstick analysis protein+ blood+

renal ultrasound normal

Which one of the following investigations would be most appropriate for this patient?

1 ) 24 hour urinary protein estimation

2 ) Measurement of anti-glomerular basement membrane (anti-GBM) antibodies

3 ) Measurement of anti-neutrophil cytoplasmic antibodies (ANCA)

4 ) plasma protein electrophoresis

5 ) renal angiography

71) A 55 year old woman presents with a non-pruritic rash that had developed over the last two months. Examination revealed several, circular, erythematous, raised, smooth-surfaced lesions of variable size from 1 – 5 cms in diameter on the elbows, extensor aspects of the forearms and knuckles.What is the most likely diagnosis?

1 ) eczema

2 ) granuloma annulare

3 ) psoriasis

4 ) tinea corporis

5 ) urticaria

72) A 17 year-old male student presents with a three week history of thirst, polyuria, balanitis and weight loss. What is the most appropriate next investigation?

1 ) 75 g glucose tolerance test

2 ) Fructosamine concentration

3 ) HbA1c

4 ) Random plasma glucose concentration

5 ) Urinary ketones

73) Whilst being investigated for infertility, a 30-year-old woman is noted to have some bruising on her limbs with a palpable spleen on abdominal examination. Investigations reveal:

Haemoglobin 10. 0 g/dL (11. 5 - 16.5)

White cell count 110 x 109/L (4 - 11)

Neutrophils 60 x 109/L (1.5 -7)

Lymphocytes 2 x 109/L (1.5 - 4)

Monocytes 0.8 x 109/L (0 - 0. 8)

Eosinophils 0.3 x 109/L (0. 04 - 0.4)

Basophils 0.7 x 109/L (0 - 0. 1)

Myelocytes 40 x 109/L

Myeloblasts 4 x 109/L

Platelet count 900 x 109/L (150 - 400)

What is the most likely diagnosis?

1 ) Acute myeloid leukaemia

2 ) Acute promyelocytic leukaemia

3 ) Chronic myeloid leukaemia

4 ) Essential thrombocythaemia

5 ) Myelofibrosis

74) Which one of the following organs is in direct contact with the anterior surface of the left kidney, without being separated from it by peritoneum?

1 ) Duodenum

2 ) Jejunum

3 ) Pancreas

4 ) Spleen

5 ) Stomach

75) A 60 year old male presents with bruising and tiredness. Examination reveals 4 finger breadth splenomegaly and his results reveal:

Haemoglobin 11 g/dl (11.5-16)

White cell count 100x109/l

Platelets 900x109/l

Blood film reveals a neutrophilia, basophilia, numerous myelocytes and 4% myeloblasts

Which of the following is likely to be present in this patient?

1 ) BCR-ABL gene fusion

2 ) Deletion chromosome 13

3 ) Deletion 11q13

4 ) Normal chromosomal analysis

5 ) Translocation 9;22

76) A 35-year-old woman with a five year history of treated hypothyroidism, presented following an episode of vomiting and collapse. There was a short history of weight loss. On examination she had a temperature of 37.7C, a blood pressure of 80/40 mmHg and vitiligo. Which one of the following, given intravenously, would be the most appropriate initial management?

1 ) 10% dextrose infusion

2 ) Cefotaxime

3 ) Fludrocortisone

4 ) Hydrocortisone

5 ) tri-iodothyronine

77) A 30 year-old female presents with a one year history of galactorrhoea. She has being receiving treatment for hay fever, depression, obesity and dyspepsia. Her investigations reveal:

Full Blood Count normal

Urea and electrolytes normal

Prolactin 820 mU/L (< 360)

free thyroxine (T4) 18.3 pmol/L (10-22)

TSH concentration 2.1 mU/L (0.4 - 5)

Which one of the following drugs is most likely to explain these findings?

1 ) Astemizole

2 ) Metoclopramide

3 ) Paroxetine

4 ) Ranitidine

5 ) Sibutramine

78) A 62 year old man seeks an opinion due to a tremor mostly affecting the right hand but also latterly the left hand which has gradually deteriorated over 5 years. Past medical history includes asthma for which he takes inhaled salbutamol, Hypertension for which he takes bendrofluazide and lisinopril and depression for which he takes amitriptyline. He smoked 10 cigarettes daily and drank approximately 15 units of alcohol weekly, noting an improvement in his tremor following alcohol. Examination revealed some mild titubation and a postural tremor in both arms with no worsening during finger-nose testing. What is the most likely diagnosis?

1 ) Benign essential tremor

2 ) Hyperthyroidism

3 ) Parkinson's disease

4 ) Physiological tremor

5 ) Salbutamol-induced tremor

79) A 55 year old female who received radioactive iodine over five years ago presents for annual thyroid function assessment. She is well and takes no medication.

Her results reveal:

Free Thyroxine 13.2 pmol/l (9.8 - 23)

TSH 16 mU/l (0.5-4.5 mU/l)

Total cholesterol 6.8 mmol/l(<5 mmol/l)

Plasma triglycerides 2.2 mmol/l(<2 mmol/l)

What is the most appropriate treatment for this patient's dyslipidaemia?

1 ) Cholestyramine

2 ) Fibrate therapy

3 ) Hormone replacement therapy

4 ) Statin therapy

5 ) Thyroxine

80) A 42-year-old female presents with tiredness. Her investigations reveal:

Haemoglobin 7.8 g/dl (11. 5 - 16.5)

MCV 72 fL (80 - 96)

white cell count 7.6 x 109/L (4 - 11)

platelet count 350 x 109/L (150 - 400)

serum ferritin 8 &micro;g/L (15 - 300)

She was commenced on oral iron therapy and one month later her haemoglobin concentration was 8. 0 g/dl. What is the most likely cause of the failure of her haemoglobin to respond to this treatment?

1 ) coeliac disease

2 ) folate deficiency

3 ) inadequate dosage of iron

4 ) poor compliance with therapy

5 ) sideroblastic anaemia

81) Which of the following is a polygenic disorder?

1 ) Ankylosing spondylitis

2 ) Erythropoietic porphyria

3 ) Fragile X syndrome

4 ) Huntington's disease

5 ) Pendred’s syndrome

82) An 80 year old male presented with acute right-sided weakness. Examination revealed minimal right facial weakness, impaired elevation of the right shoulder, with relatively preserved right hand strength. There was global weakness in the right leg which appeared to be maximal in the foot. Which of the following arteries is most likely to have been affected?

1 ) Anterior cerebral artery

2 ) Lenticulostriate artery

3 ) Middle cerebral artery

4 ) Posterior cerebral artery

5 ) Posterior communicating artery

83) A 79 year old female suffers a fracture neck of femur following a fall at home. Investigations are normal but her X-ray shows the bones to be rather 'thin'. It is assumed that she is osteoporotic and she is started on alendronate therapy. Which of the following is correct concerning this drug.

1 ) Enhances vitamin D action on bone

2 ) Increases absorption of calcium

3 ) Increases osteoblast activity

4 ) Increases the action of oestrogen on bone

5 ) Inhibits osteoclast activity

84) An 18 year old, woman attends antenatal clinic 12 weeks into her pregnancy where the doctor incidentally notes numerous small lumps over her trunk and freckles in her axillae. She reported that none of her relatives had any similar features.

What is the most likely diagnosis?

1 ) acanthosis nigricans

2 ) dysplastic naevus syndrome

3 ) mastocytosis

4 ) neurofibromatosis

5 ) tuberous sclerosis

85) These are the blood gas results obtained in a 20 year old female admitted to hospital.

hydrogen ion concentration 35 nmol/L (35 – 45)

pH 7.45 (7.35-7.45)

pC02 6.8 kPa (4.6 – 5.9)

bicarbonate 32 mmol/L (22 - 26)

Which one of the following is a recognised cause of this acid-base disorder?

1 ) Amitriptyline overdose

2 ) Cushing's syndrome

3 ) hepatic failure

4 ) pregnancy

5 ) salicylate poisoning

86) An 18 year old woman sustains severe head injuries in a road traffic accident The following day her investigations show:

Sodium 160 mmol/L (133-145)

Chloride 120 mmol/L (100 – 112)

Urea 3.0 mmol/L (3 –7)

Creatinine 90 micromol/L (50 – 100)

Which one of the following statements is correct?

1 ) Rapid rehydration with 5% dextrose is indicated

2 ) She should be treated with sodium restriction

3 ) She has the syndrome of inappropriate antidiuretic hormone secretion (SIADH)

4 ) She will have a hypercholraemic acidosis

5 ) Urine osmolality will be low

87) A previously well 40 year old man is admitted with a single haematemesis after taking 300 mg of aspirin five hours previously. On examination, pulse was 120/min with a blood pressure of 110/75 rmHg (lying) and 90/60 mmHg (standing). Respiratory and abdominal examination was otherwise normal.

His haemoglobin concentration returned as 7 g/dL (13.0 - 16.5. What is the most likely cause of his haemetemesis?

1 ) Angiodysplasia

2 ) Duodenal ulcer

3 ) Gastric cancer

4 ) Gastric erosions

5 ) Oesophagitis

88) A 40-year-old female who has been prescribed thyroid replacement therapy has routine thyroid function tests. On examination she appeared clinically euthyroid with no abnormal findings. Her TFTs revealed:

TSH 3.2 mU/L (0. 35 - 5.0)

Total T4 20 nmol/L (55 - 144)

free T4 2.6 pmol/L (9 - 24)

Total T3 2.5 nmol/L (0. 9 - 2.8)

Which one of the following statements is correct?

1 ) Her thyroid hormone replacement is adequate

2 ) Investigation of pituitary function is required

3 ) She has tertiary hypothyroidism

4 ) She has a thyroiditis

5 ) She has sick euthyroid syndrome

89) A 25 year old female is admitted with acute dyspnoea and chest pain. A diagnosis of pulmonary embolism is confirmed and her investigations reveal urine dipstick protein ++ but no blood, anti-double standed DNA antibodies of 200 U/mL (0 - 73), with a 24 hour urinary protein concentration of 5g (< 0.2). Which one of the following diagnoses is most likely to be found on renal biopsy?

1 ) AA amyloid

2 ) Focal segmental glomerulonephritis

3 ) IgA nephropathy

4 ) membranous nephropathy

5 ) minimal change nephropathy

90) A 56 year old female is referred to clinic by her GP who notes hepatomegaly. Six years ago she was diagnosed with diabetes mellitus and takes metformin 500 mg tds and gliclazide 80mg bd. She drinks approximately 15 units of alcohol weekly amd stopped smoking 10 years ago.

On examination she has a BMI of 36.2 kg/m, no stigmata of liver disease are evident but she has 6 cm hepatomegaly.

Investigations disclose:

Total bilirubin 11 micromol/L (1 - 22)

Alkaline phosphatase 145 U/L (45 - 105)

AST 100 U/L (1 -31)

ALT 150 U/L (5 - 35)

Albumin 40 g/L (37 - 49)

Ferritin 434 mg/L (15 - 300)

Ultrasound of the abdomen reveals an echobright appearance of the liver and gallstones in the gallbladder.

What is the most cause of her liver disease?

1 ) Alcoholic liver disease

2 ) Drug induced hepatitis

3 ) Gallstone disease

4 ) Haemochromatosis

5 ) Non-alcoholic steatohepatitis (NASH)

91) A 69-year-old woman admitted for a surgical procedure is noted to have a soft systolic murmur at the left sternal edge. Her ECG and chest X-ray were normal and transthoracic echocardiography revealed a small posterior pericardial effusion with normal valves. Which of the following would be the most appropriate next step in this patient’s management?

1 ) A diagnostic Pericardial aspiration

2 ) mammography

3 ) purified Protein derivative test for tuberculosis

4 ) reassurance

5 ) right heart catheterisation

92) A 25 year old male with learning difficulties presents with behavioural problems. He confessed to smoking the occasional cannabis joint. Which of the following is most likely to be the cause of his behavioural problems?

1 ) cannabis

2 ) Dementia

3 ) Depression

4 ) Mania

5 ) schizophrenia

93) A post-marketing surveillance study of a new heart failure therapy to the market was carried out on 10,000 subjects who had completed clinical trials. Which one of the following most accurately reflects the information generated from such a study?

1 ) Adverse events profile

2 ) Cost benefit analysis

3 ) Cost effectiveness

4 ) Comparative therapeutic efficacy

5 ) Drug potency

94) A 72-year-old male presented to his GP with depression after the death of his wife. His notes also reveal that he has a two-year history of urinary hesitancy and poor stream. His GP prescribed him some medication and the following day he developed acute urinary retention. Which of the following drugs is most likely to have precipitated the urinary retention?

1 ) Amitriptyline

2 ) Diazepam

3 ) Fluoxetine

4 ) Venlafaxine

5 ) Zopiclone

95) A 68 year old male is admitted with a two months history of difficulty raising his arms, ascending stairs, and is also aware of a dry mouth. He smokes 15 cigarettes daily and admits to heavy alcohol consumption. On examination he has proximal weakness affecting all four limbs with absent tendon reflexes. His chest X-ray shows a right pleural effusion What is the most likely diagnosis?

1 ) alcohol induced myopathy

2 ) Eaton- Lambert syndrome

3 ) myasthenia gravis

4 ) polymyalgia rheumatica

5 ) polymyositis

96) You are asked to provide advice on a 35 year old woman who is admitted under the maxillo-facial surgeons for extraction of wisdom teeth. The only concern was that she had developed prolonged bleeding following a tooth extraction 10 years previously and had required had required suturing. Besides this, she gave no other history of bleeding. What is the most likely diagnosis?

1 ) factor IX deficiency

2 ) factor V Leiden

3 ) factor XII deficiency

4 ) primary antiphospholipid syndrome

5 ) von Willebrand's Disease

97) A 68 year-old woman with atrial fibrillation is admitted for DC cardioversion. The procedure resulted in successful restoration of sinus rhythm. Which one of the following drugs would be most likely to maintain sinus rhythm following this procedure?

1 ) amiodarone

2 ) digoxin

3 ) diltiazem

4 ) sotalol

5 ) verapamil

98) A 70 year old female who has a history of chronic anxiety presents with a 3 day history of severe left temporal headache radiating from the eye to the scalp. She had also experienced discomfort during eating.

Which one of the following drugs should be given to this patient while awaiting the results of diagnostic tests?

1 ) Acyclovir

2 ) Carbamazepine

3 ) Diclofenac

4 ) Prednisolone

5 ) Sumatriptan

99) A study comparing contrast CT colonography with the reference technique of colonoscopy for large bowel carcinoma reveals the following data in 400 patients:

Investigation CT Positive CT Negative

Colonoscopy Positive 30 10

Colonoscopy negative 20 340

Which one of the following most accurately describes the performance of CT versus colonoscopy for the diagnosis of large bowel cancer?

1 ) There are 340 false negatives

2 ) There are 370 false negatives

3 ) There are 10 false positives

4 ) There are 20 false positives

5 ) There are 20 true negatives

100) A 50-year-old woman is referred with a two-week history of difficulty walking and weakness in her arms. On examination, there was proximal and distal limb weakness which is more marked in the legs than the arms. All tendon reflexes were absent and the plantar responses were flexor. There was no sensory loss. Blood pressure in the supine position was 140/78 mmHg (lying) and was 110/70 mmHg on standing. What is the most likely diagnosis?

1 ) polymyositis

2 ) cervical cord compression

3 ) Guillain-Barr&eacute; syndrome

4 ) myasthenia gravis

5 ) poliomyelitis

101) A 40 year-old man is referred with gastro-oesophageal reflux disease (GORD). Which of the following concerning GORD is correct?

1 ) Acid suppressant therapy should not be given continuously

2 ) Endoscopy is mandatory

3 ) In the presence of Barrett's oesophagus, the risk of future malignancy can be assessed endoscopically without biopsy

4 ) Oesophageal pH monitoring is a good guide to therapy

5 ) Symptoms do not correlate with mucosal status at endoscopy

102) An 80 year-old male presented with palpitations of 5 hours duration. One month previously he suffered weakness of the right arm and problems with his speech which resolved within 4 hours. He was taking no medication. On examination, he was stable with a pulse of 135 beats per minute which was confirmed to be atrial fibrillation on ECG. He had a blood pressure of 112/80 mmHg, appeared clinically euthyroid. Within one hour he reverted to sinus rhythm spontaneously. Echocardiogram was normal but a 24 hour ECG revealed three episodes of atrial fibrillation each lasting around ten minutes. Which one of the following is the most appropriate initial treatment for this patient?

1 ) Amiodarone

2 ) Aspirin

3 ) atenolol

4 ) digoxin

5 ) warfarin

103) An 18 year-old female is admitted with a depression of her conscious level. Arterial blood gas analysis revealed:

pH 7.26

pO2 12.1 kPa

pC02 3.9 kPa

standard bicarbonate 14.7 mmol/L

Which one of the following would account for these results?

1 ) Analytical error

2 ) Metabolic acidosis

3 ) Persistent vomiting

4 ) Respiratory acidosis

5 ) Respiratory alkalosis

104) A 50 year old business man who has been drinking heavily for at least two years, states that he drinks alcohol on his way into work as he suffers from anxiety attacks. Which one of the following statements is true regarding these episodes?

1 ) they are imagined

2 ) they are not accompanied by tremor

3 ) they are still present after drinking

4 ) they will improve with three weeks of abstinence from alcohol

5 ) they will worsen with three weeks of abstinence from alcohol

105) A 70 year old woman is referred by a GP to the surgeons with a breast lump. She was asymptomatic but her investigations reveal:

Corrected calcium 2.72 mmol/L (2.2 - 2.6)

Phosphate 0.80 mmol/L (0.8-1.4)

Alkaline phosphatase 110 U/L (20 - 95)

PTH concentration 5.1 pmol/L (0.9-5.4)

What is the most likely diagnosis?

1 ) bony metastases

2 ) chronic vitamin D excess

3 ) ectopic PTH related peptide (PTHrp) secretion

4 ) multiple myeloma

5 ) primary υπερπαραθυρεοειδισμός

106) A 70 year old woman presented with an acute, severe occipital headache, unsteadiness of her gait and vomiting. She had a history of poorly controlled hypertension. On examination there was nystagmus to the left, ataxia of the left limbs and gait ataxia.

What is the most likely diagnosis?

1 ) acute cerebellar haemorrhage

2 ) basal ganglia haemorrhage

3 ) pontine haemorrhage

4 ) subdural haemorrhage

5 ) temporal lobe haemorrhage

107) A 55-year-old nurse developed bronchospasm and urticaria twenty minutes into surgery under general anaesthesia. The mast cell tryptase concentration confirmed an acute allergic reaction. Later, it transpired that she had developed allergic reactions at her dentist and had developed frequent episodes of wheezing when assisting at sterile procedures. What is the most likely diagnosis?

1 ) allergy to anaesthetic induction agents

2 ) allergy to local anaesthetic agents

3 ) Latex allergy

4 ) pressure urticaria

5 ) systemic mastocytosis

108) A 25-year-old male presents to casualty with weakness of his right hand. Examination reveals weakness of right wrist and finger extension. What is the most likely diagnosis?

1 ) axillary nerve palsy

2 ) C8 nerve root lesion

3 ) proximal median nerve lesion

4 ) radial nerve lesion

5 ) ulnar nerve lesion at the elbow

109) A 70 year old man presented with increasing dyspnoea. In his history, he had suffered a myocardial infarction two years previously which had been complicated by ventricular arrhythmias. At admission his oxygen saturations were 85% on air and a chest X-ray revealed bilateral patchy infiltration of both lung fields with a cardiothoracic ratio of 20/30 cm. Which of the following drugs that he has been prescribed is most likely to explain these findings?

1 ) Amiodarone

2 ) Atorvastatin

3 ) Aspirin

4 ) Frusemide

5 ) Ramipril

110) A 60 year old female presents with recent-onset dyspnoea and noisy breathing. Her chest X-ray showed right deviation of the trachea due to a retrosternal goitre. Which of the following tests is most useful in the assessment of airflow obstruction due to the goitre?

1 ) flow volume curve

2 ) forced expiratory flow volume in one second

3 ) forced vital capacity

4 ) peak expiratory flow rate

5 ) residual volume

111) A 63-year-old woman presents following a visit to the well woman clinic where she is noted to be hypertensive. She has a past history of hip osteoarthritis for which she has taken regular paracetamol. On examination she is obese with a BMI of 35 (<25), has a blood pressure of 180/100 mmHg and glycosuria is noted.

Her Investigations show:

Fasting plasma glucose 18.3 mmol/L

Serum urea 9.8 mmol/l

serum creatinine 129 micromol/L

24 hour urine protein concentration 1.8 g/d

Normal ultrasonic appearances of both kidneys

Which of the following is the most likely diagnosis?

1 ) Analgesic nephropathy

2 )Chronic glomerulonephritis

3 ) Diabetic nephropathy

4 ) Hypertensive nephropathy

5 ) Ischaemic nephropathy

112) A 68 year old female with terminal bowel cancer is receiving optimal doses of morphine sulphate therapy. Which of the following effects may be expected with the addition of a partial opioid agonist?

1 ) increased analgesia

2 ) increased respiratory depression

3 ) increased sedation

4 ) no change

5 ) reduced analgesia

113) A 76 year old male attends clinic with his wife who states that her husband has become disinterested and withdrawn.

Which of the following would favour a diagnosis of dementia rather than depression?

1 ) Agitation

2 ) Poor short term memory

3 ) Reduced libido

4 ) Self-reported concern of poor memory

5 ) Urinary incontinence

114) A 72 year old man noted to have a systolic murmur undergoes an echocardiogram which demonstrates aortic stenosis. Which of the following is associated with a poor prognosis in this patient?

1 ) Aortic regurgitation

2 ) Cardiomegaly on chest X-ray

3 ) Clinical features of left ventricular failure

4 ) ECG evidence of left ventricular hypertrophy

5 ) severe valvular calcification on echocardiogram

115) A 79 year-old male is admitted with acute confusion. His relatives who accompany him assert that he had been entirely self-caring, does not drink alcohol and was taking no previous medication. He was diagnosed with a urinary tract infection and commenced antibiotics but remained agitated and distressed. Which ONE of the following treatments is the most appropriate for his agitation?

1 ) chlorpromazine

2 ) Diazepam

3 ) Haloperidol

4 ) Temazepam

5 ) Trazadone

116) A paper describes a new diagnostic test for myocardial infarction. You want to know what proportion of patients who are classified as not having had a myocardial infarction by the test will actually not have had a myocardial infarction. Which one of the following measurements would indicate this?

1 ) accuracy

2 ) negative predictive value

3 ) positive predictive value

4 ) sensitivity

5 ) specificity

117) An 18 year-old woman presents with an acute pulmonary embolism in the ninth week of pregnancy.

What is the most appropriate treatment for this patient throughout her pregnancy?

1 ) Aspirin

2 ) Intravenous unfractionated heparin

3 ) Subcutaneous low molecular weight heparin

4 ) Subcutaneous unfractionated heparin

5 ) Warfarin

118) A publication assesses a new diagnostic test for thyroid cancer. Which of the following terms would reflect the number of cases of thyroid cancer correctly identified by this new test?

1 ) accuracy

2 ) negative predictive value

3 ) positive predictive value

4 ) Sensitivity

5 ) Specificity

119) A 20 year old female with cystic fibrosis presents in early pregnancy wanting advice. Genetic analysis reveals that her partner is a carrier of the cystic fibrosis gene. What is the chance of her child having cystic fibrosis?

1 ) 10

2 ) 25

3 ) 50

4 ) 75

5 ) 100

120) A 50-year-old female presented with a week’s history of pain and stiffness in her shoulders and wrists with symptomatic deterioration in the morning. On examination, there was synovitis of both wrists and there was no proximal muscle wasting or weakness. Her ESR was 50 mm/hr (0 - 20). What is the most likely diagnosis?

1 ) polymyalgia rheumatica

2 ) polymyositis

3 ) reactive arthritis

4 ) rheumatoid arthritis

5 ) systemic lupus erythematosus

121) A 26 year old female is referred with intermittent diarrhoea present for many years. She states that her weight has been steady but describes watery motions up to six stools per day and has also noted abdominal discomfort with bloating. She has not been aware of any blood in the motions or melaena. She describes no other medical history and denies taking any medication.

Investigations show:

Full Blood count Normal

Urea and electrolytes Normal Albumin 39 g/L (37 - 49)

Corrected calcium 2.2 mmol/L (2.2-2.6)

Alkaline phosphatase 94 U/L (45 - 105)

C-reactive protein 6 mg/L (< 10)

Prothrombin time 12 s (11.5 -15.5)

What is the most likely diagnosis?

1 ) Crohn's disease

2 ) intestinal tuberculosis

3 ) lactose intolerance

4 ) laxative abuse

5 ) microscopic colitis

122) With regard to cardiac troponins, which ONE of the following statements is correct?

1 ) Elevated plasma troponin concentrations are specific markers of ischaemic heart disease

2 ) Plasma troponin concentrations are typically elevated three weeks after

Link to comment
Μοιράσου σε άλλους δικτυακούς τόπους

124) A 68-year-old male is referred by his general practitioner with deteriorating hypertension and renal function. His investigations disclose:

serum creatinine 250 micromol/L (60 - 110)

urinalysis + protein

renal ultrasound examination:

• left kidney 9cm long

• right kidney 7cm, no obstruction (10 - 12cm)

Which of the following would be the most appropriate investigation for this patient?

1 ) intravenous renography

2 ) isotope renography

3 ) MR angiography

4 ) renal biopsy

5 ) retrograde pyelography

ANSWERS

1) 3

Approximately 50% of subjects with FSGS do not respond to steroid therapy but ACE inhibitors are a recognized strategy to slow the progression of renal disease. This patient is clearly at high risk of cardiovascular disease with a very high cholesterol but the question specifically asks about renal disease.

2) 3

This elderly patient presenting with bone pains has elevated alkaline phosphatase with normal calcium concentrations suggesting a diagnosis of Paget's. The slightly elevated PSA is in keeping with benign prostatic hypertrophy rather than prostatic malignancy where a PSA of above 10 would be expected. The normal calcium and only slightly elevated ESR argues against osteomalacia, myeloma and PMR.

3) 3

This gent is intoxicated. He has a normal acid base balance slight hyponatraemia reflecting dilution and very high osmolality reflecting the presence of ethanol. Methanol would produce an acidosis. Diazepam is not an osmolyte nor would the other agents produce this picture.

4) 4

Laparoscopic fundoplicationis the treatment of choice for patients with GORD refractory to or intolerant of, Proton Pump Inhibitor therapy. The patient should have had an endoscopy at least 6 months prior to surgery to exclude any unsuspected pathology – Barrett’s oesophagus or adenocarcinoma. An oesophageal transit study is indicated to rule out a primary motor disorder (eg achalasia, scleroderma) when suspected and to rule out aperistalsis, which may result in post-operative dysphagia after some forms of fundoplication.

5) 5

Anaplastic thyroid cancer carries a very poor prognosis with the vast majority (~90%) having local invasion (cervical lymph glands) and local infiltration, (particularly the trachea) at diagnosis. Lung and bone metastases are common at presentation ~ 50%. Upper airways obstruction frequently requires tracheostomy.

6) 3

In a normal healthy person's scalp about 85% of the hair follicles are actively growing hair and 15% are If there is some shock to the system, as many as 70% of the scalp hairs can be precipitated into a resting state, thus reversing the usual ratio. Typical precipitants include illnesses, operations, accidents and childbirth. The resting scalp hairs, now in the form of club hairs, remain firmly attached to the hair follicles at first. It is only about 2 months after the shock that the new hairs coming up through the scalp push out the "dead" club hairs and increased hair fall is noticed. Thus, paradoxically, with this type of hair loss, hair fall is a sign of hair regrowth.

As the new hair first comes up through the scalp and pushes out the dead hair a fine fringe of new hair is often evident along the forehead hairline. At first the fall of club hairs is profuse and a general thinning of the scalp hair may become evident but after several months a peak is reached and hair fall begins to lessen, gradually tapering back to normal over 6-9 months. As the hair fall tapers off the scalp thickens back up to normal, but recovery may be incomplete in some cases.

7) 4

This asymptomatic patient requires only lifestyle advice i.e weight loss if appropriate, an appropriate diet and reduction of alcohol.

8) 1

The differential diagnosis of cavitating lung lesions is shown below. The most likely diagnosis in this non-smoking man is post primary tuberculosis as a result of reactivation of quiescent disease. He has several risk factors including increasing age and diabetes. The patient is unable to produce sputum therefore undertaking a bronchoscopy with bronchial washings for microscopy staining and culture is the investigation of choice. Gastric lavage for AFB is unpleasant for the patient has a lower yield than bronchoscopy and is therefore rarely undertaken now. Causes of Cavitating Masses on CXR: (a) Lung abscess (B) Tuberculosis © Fungal infection (eg. Histoplasmosis, Cocccidioidomycosis) (d) Malignancy (e) Wegener’s Granulomatosis ) commonly (f) Rheumatoid Arthritis ) multiple (g) Infarction )

9) 5

Idiopathic sideroblastic anaemia is a member of the myelodysplastic syndromes. In this condition, the red blood cells are normal or macrocytic and there is anisocytosis and poikilocytosis on the peripheral films. This isn't iron deficiency anaemia as the ferritin is normal and a microcytic anaemia would be expected. A normochromic normocytic anaemia is expected in renal disease (epo deficiency). Hypothyroidism may cause a slight elevation of MCV but no anisopoikilocytosis is evident.

10) 2

This patient is likely to have Bulimia – young girl with a likely low body mass contributing to the low urea, vomiting contributing to the hypokalaemia/hyponatraemia. Her urine sodium is appropriately low and due to a relative dehydration she has appropriately concentrated urine. This is not Addison’s disease as urine sodium would be high with high urea and likely high potassium. Similarly it is not SIADH due to the low urine sodium. Diuretic abuse would cause high urine sodium. Water intoxication would produce a dilute urine.

11) 1

The most appropriate therapy advocated by the National Osteoporosis Society for the prevention of steroid-induced Osteoporosis would be bisphosphonate therapy such as Didronel or alendronate. These are the only class of drug shown to offer osteoprotection with steroid therapy. Patients taking 7.5 mg or more of prednisolone daily for 3 months or longer should be offered osteoprotection. HRT would not really be appropriate for this subject who is ten years past the menopause and likely to be free of all menopausal symptoms.

12) 4

This prevention study for stroke reveals that 20 patients need to be treated to prevent one event. Thus if you treat a 1000 patients then you will expect to have 50 fewer strokes.

13) 1

Approximately 4mg of iron circulate within the plasma with a total body iron store of 3-4 g (2500 mg in the RBCs, 500mg in liver, 500 mg in macropahages and about 500 mg in muscle). From an intake of approx 6mg/1000kcal of dietary iron only 15% is bioavailable. The majority of iron contained within the RBCs is metabolised and re-utilised but 1mg per day is lost through the gut. Ferritin, the plasma protein responsible for binding iron is an acute phase reactant protein and increases in inflammatory conditions and following surgery. Transferrin is a glycoprotein responsible for internal ion exchange and the content within mucosal cells is naturally low in haemochromatosis with high saturation.

14) 2

This patient has hyponatraemia and hypokalaemia. This is probably due to the bendrofluazide which should be stopped. It is unlikely to be SIADH ,of which tolbutamide is a cause, as the hypokalaemia is not typically associated.

Her hyperglycaemia with an osmotic diuresis would cause dehydration. Addison’s disease would be associated with hyperkalaemia, hypotension and elevated urea.

15) 5

Achondroplasia, APKD, C1 esterase deficiency (hereditary angio-oedema) and FH are usually inherited as autosomal dominant traits. Friedreich's ataxia is characteristically an autosomal recessive inheritance.

16) 4

This man has Bronchiectasis as evidenced by his regular production of sputum associated with breathlessness, his repeated lung infections and the signs of bilateral inspiratory crackles. Retained mucus is the most important reason why bronchiectatic patients become infected. Postural drainage is therefore the cornerstone to treating bronchiectasis and should be undertaken at least once per day and more frequently during exacerbations. There have been trials looking at regular antibiotic therapy versus symptomtic treatment in patients with cystic fibrosis colonised with pseudomonas (eg Elborn JS et al. Thorax 2000;55: 355-358) but there is currently no evidence that this approach is of benefit in bronchiectasis. Similarly inhaled corticosteroids should not be used routinely in bronchiectasis until further evidence of their effect on lung function and exacerbation frequency is available. Surgical resection as a curative proceedure can be performed for localised disease when underlying causes such as primary ciliary dyskinesia have been excluded. In this patient the bilateral crackles suggests widespread disease.

17) 2

Scattered microaneurysms signify background DRn. IRMAs and soft exudates signify pre-proliferative retinopathy. According to the NleH, urgent referral to an ophthalmologist (seen within one week) is required if there is proliferative retinopathy or there is evidence of clinically significant macular oedema (hard exudates at the fovea).

18) 2

Patients with potentially operable tumours that are either too unfit for surgery or refuse surgery may be suitable for radical radiotherapy i.e. radiotherapy with intention to cure using doses of at least 60 Gy. Using continuous hyperfractionated accelerated radiotherapy (CHART) can improve 2 year survival significantly from 20% to 29% compared with conventional radiotherapy (Saunders M et al. Lancet 1997 350: 161-165). Contraindications are tumours larger than 4cm and poor pulmonary function (generally taken as FEV1 less than 50% predicted).

19) 4

Metformin is a biguanide which acts to improve insulin sensitivity through mechanisms that involve hepatic gluconeogenesis and improved muscle glucose utilization. Thus, some insulin must be produced for it to have an effect. It is associated with hypoglycaemia although this side effect is unusual. It is contra-indicated in subjects with renal failure, hepatic failure and heart failure due to the association with lactic acidosis.

20) 1

This patient should be treated with ACEi as he has microalbuminuria. ACEis have been shown to slow the progression of diabetic nephropathy. Otherwise his glycaemic control is excellent. Dietary protein restriction would be more relevant for chronic renal failure.

21) 4

This patient has a mild metabolic alkalosis with what appears to be respiratory compensation as reflected by the elevated pCO2. Amitriptyline overdose is associated with acidosis as is salicylate poisoning. Hepatic failure usually presents with acidosis. This type of picture is associated with prolonged vomiting (as in pregnancy), diarrhoea, diuretic therapy and in Cushing’s syndrome or in those receiving high dose corticosteroids. With no other information provided for this case, common things being common, one should select pregnancy as the best answer for a 20 year old female.

22) 4

Typically, a small VSD generates a pansystolic murmur at the LSE accompanied by a thrill. The murmur may be heard at the apex but is usually loudest at the LSE. Very small defects may generate an early/late systolic murmur. Fixed splitting of the heart sounds usually accompanies atrial septal defects.

23) 3

The diagnosis is EBV infection, Infectious Mononucleosis, which may be confirmed by the presence of IgM to EBV.

24) 1

The diagnosis is amyloidosis which causes an infiltrative restrictive cardiomyopathy typically in patients of this age group. Other causes include sarcoidosis, radiotherapy, systemic sclerosis and carcinoid syndrome. Cocksackie produces a viral myocarditis with the likelihood of a dilated appearance on echo. Marfan’s is likely to cause valvular regurgitant defects and a dilated cardiomyopathy. Down’s syndrome is more likely to be associated with AV canal defects and consequent dilatation. Turner’s syndrome is associated with atrial septal defects and co-arctation and bicuspid valvular defects.

25) 4

Episodic cough and wheeze with nocturnal symptoms are classical of asthma. Occupational asthma is the commonest industrial lung disease with over 400 causes and accounts for up to 10% of adult onset asthma. The commonest occupations affected are spray painters, Bakers, Chemical processors, plastics workers and welders and soldering. Patients are characteristically better when on holiday. The diagnosis is confirmed by serial PEF measurements at home and at work. Recordings should be performed 2 hourly for 4 weeks or if this is not possible metacholine/histamine challenges can be undertaken after days at work and away from work. Following objective confirmation of the diagnosis the underlying cause should be identified.

26) 3

Alcohol is mainly metabolized in the liver to acetaldehyde by alcohol dehydrogenase. Acetaldehyde is then oxidized to acetate by acetaldehyde dehydrogenase (AcDH). Disulfiram irreversibly inhibits the oxidation of acetaldehyde by competing with the cofactor nicotinamide adenine dinucleotide (NAD) for binding sites on (AcDH). The increased acetaldehyde levels are thought to produce the unpleasant side effects associated with acetaldehyde syndrome such as headaches, nausea, flushing etc.

27) 4

HbsAg is positive, Hepatitis E antigen is negative and the DNA being undetectable suggests that it is Chronic HBV infection with low viral replication.

28) 4

The most likely explanation in this patient with a prior inferior myocardial infarct is mitral valve prolapse due to papillary muscle rupture.

29) 5

The most appropriate strategy for secondary prevention would involve further blood pressure reduction with an ACEi which would not only reduce CV risk as suggested by the HOPE study but also reduce microvascular risk as revealed by UKPDS. The NCEP ATPIII criteria suggest a cholesterol less than 4 and this patient already has a low and would not benefit as much from the addition of a statin. The increase of aspirin from 150 to 300 mg would offer no added advantage. Orlistat is used under specific criteria for weight reduction and has, as yet, not been shown to reduce CV risk in T2DM. There’s no reason here for a 24 hr tape.

30) 1

The features of shoulder pain associated with a past history of carpal tunnel syndrome in a patient receiving haemodialysis suggests a diagnosis of b2 microglobulin amyloidosis. Amyloid deposits composed of b2-microglobulin as the major constituent protein are mainly localized in joints and periarticular bone and lead to destructive arthropathy which tends to develop 5 to 10 years after the initiation of dialysis. Death from amyloidosis of gut and heart may occur after 20 years of dialysis.

31) 5

This patient has a metabolic alkalosis with type 1 respiratory failure as evidenced by low pO2 and low pCO2. Chronic venous thromboembolism would be the most likely explanation for this man’s presentation. Hyperventilation would be excluded by the type 1 respiratory failure, an inhaled foreign body would not produce such a picture and an atypical pneumonia would be associated with pyrexia and some clinical signs. The differential diagnosis here is pulmonary fibrosis but basal crackles may be expected and the history is somewhat short.

32) 5

An azygous lobe is seen in about 0.5% of routine Chest X-rays and is a normal variant. It is seen as a 'reverse comma sign' behind the medial end of the right clavicle.

33) 3

This is a typical presentation of pseudo-gout / Calcium pyrophosphate arthropathy with evidence of osteoarthritis, calcification of the articular cartilage and no growth on culture. The differential does include gout but there is nothing else within the history to suggest this as the diagnosis. Distinguishing between the two depends on analysis of the crystals with CPP crystals demonstrating a positive birefringence and urate crystals demonstrating a negative birefringence.

34) 2

The patient is diabetic as reflected by the elevated random glucose, has hyponatraemia, renal impairment and mild hyperkalaemia which appears to have been precipitated following diarrhoea. Hypoadrenalism is unlikely as he has hyperglycaemia. SIADH is unlikely in the context of the elevated urea and potassium. The high potassium would argue against this being caused by the bendrofluazide. Similarly, hyperglycaemia would itself cause a picture of dehydration where the sodium is likely to be elevated.

35) 1

The features of relief with alcohol, postural tremor and a slight and rather benign deterioration over five years suggests a diagnosis of Benign essential tremor. Physiological tremor usually affects the hands only. In this case there is titubation, again a feature of essential tremor. The mild features argue against Parkinson’s disease. The condition is typically slowly progressive and occurs from approximately 50 years.

36) 1

The patient presents with an asymptomatic eruption on his trunk. The lesions are scaly, hypopigmented and are not associated with any systemic disease. This is characteristic of pityriasis versicolor, which is caused by the unicellular yeast Pityrosporum ovale and Pityrosporum orbiculare. The yeast is lipophilic and is encouraged by an increase in environmental temperature, thus many patients notice that the condition begins after a summer vacation. It is a disorder of the healthy but the immunocompromised are at risk. The condition is asymptomatic and appears pale in comparison to the normal skin. The fungus affects the melanocytes hence the hypo pigmentation.. The treatment options include topical imidazole creams , selenium sulphide shampoo and if not responding to topical treatment oral itraconazole 200mg once a day for 7 days. In this patient the topical treatment should be tried first.

37) 3

Dental scaling is regarded as a high risk procedure and should be covered by antibiotic prophylaxis.

38) 4

The suggested diagnosis is vitiligo which is associated with numerous autoimmune conditions including in order of frequency, autoimmune hypothyroidism, pernicious anemia, alopecia areata and Addison's disease.

It is associated with both type 1 and 2 autoimmune polyendcorine syndromes but these are much rarer than the former diagnoses.

39) 1

G6PD deficiency is inherited in an X-linked fashion and predisposes RBCs to haemolysis. Drugs recognised to predispose to acute haemolysis in G6PD deficiency include antimalarials such as Primaquine, sulphonamides, Nitrofurantoin and Nalidixic acid.

40) 2

The features suggest an L5 radiculopathy which would be associated with a loss of sensation in the dorsum of the foot and big toe.

41) 2

This patient is likely to have Cushing’s syndrome. It’s a difficult choice between overnight dexamethasone suppression test and the urine free cortisol estimation but on balance, the simplest test would be Urine free cortisol assessment. 9am cortisol and ACTH concentrations will not confirm the diagnosis. A short synacthen test is used to confirm hypoadrenalism.

42) 1

This patient appears to have acute renal failure with severe acidosis, hyperkalaemia and has palpitations. The patient should be rehydrated, treated with insulin and given bicarbonate, but the immediate treatment particularly in the context of a life threatening arrhythmia would be Calcium gluconate.

43) 2

ARDS is characterised by hypoxaemia, reduced lung compliance, pulmonary hypertension and pulmonary infiltrates on the chest X-ray. There is damage to the capillary and endothelial cell linings resulting in oedema and leakage of proteins and cells into the interstitial and alveolar spaces at normal pulmonary capillary hydrostatic pressures. Wedge pressure unlike the high pressures seen with LVF and pulmonary oedema is often normal. Hypercapnia often a late feature of ARDS does not distinguish from any other cause of type 2 respiratory failure.

44) 5

Whipple's disease is caused by Tropheryma Whippeli and symptoms include chronic diarrhoea, arthralgia, pyrexia and lymphadenopathy.Diagnosis is by microscopy of Jejunal biopsy specimen which shows macrophages with PAS positive granules.

45) 2

Don’t be put off by the description of the case, these histological features are typical of Coeliac disease with villous atrophy, crypt hyperplasia/hypertrophy, inflammatory infiltrate of the lamina propria and intra-epithelial lymphocytes. Useful serology includes anti-TTG antibodies which would be expected in over 90% of cases. Treatment of this case would therefore entail gluten-free diet.

46) 3

The most relevant investigation with anyone with a red, swollen and painful joint would be joint aspiration sending off for cultures and analysis for crystals. Differential diagnoses include gout (where serum urate may fall during acute attack),pseudogout and infection. All diagnoses would be adequately addressed by joint aspiration.

47) 3

Daft question really. The most appropriate initial action would be to get the investigations done as quickly as possible – arterial blood gases and ECG as the latter may show QRS widening and merit treatment. Then, the next step would be gastric decontamination with lavage and activated charcoal. Treatment with bicarbonate is also advocated as this patient displays features of severe TCA overdose. He doesn’t need a CT scan as the symptoms are typical of tricyclic overdose. Flumazenil is not appropriate for this patient as the symptoms are mostly of TCA overdose nor is IV atenolol appropriate for the arrhthymias – bretylium, phenytoin or lidocaine.

48) 5

This patient’s blood gases show she is receiving too high a concentration of oxygen which is likely to have precipitated her hypercapnic acidosis. Patients with COPD should not in general receive more than 24-28% oxygen without arterial blood gas monitoring. Reduction of FiO2 may be sufficient to improve this lady’s acidosis. Once this is done she should be treated with nebulised B/D driven on air and if she fails to improve despite controlled oxygen and bronchodilators NIV is indicated.

49) 2

The endoscopy shows small varices with no evidence of bleeding but diffuse oozing of blood. Hence endocopic measures like banding for small varices will not be useful. There is probably no evidence that vit K is helpful as the coagulation is already likely to be deranged. Oral propranolol is useful as a later prophylaxis of variceal bleed.

50) 4

The patient presents with tense blisters on her arms, trunk and legs. She is otherwise well and there is no mucosal involvement. This is typical of bullous pemphigoid. Dermatitis herpetiformis presents with itchy excoriated areas in the elbows knees and buttocks. Erythema multiforme presents with characteristic target lesions. Herpes simplex is vesicular and in generalized cases the patient is likely to be unwell. Pemphigus presents with superficial erosions and usually there is mucosal involvement.

51) 3

Klinefelter’s syndrome and Down’s syndrome are diagnosed principally by chromosomal analysis/karyotype – XXY in the former and trisomy C21 or translocation in the latter. A trinucleotide CAG repeat expansion in the huntingtin gene is diagnostic of Huntington disease. The majority of cases of HOCM are autosomal dominantly inherited yet defective genes are located on a variety of chromosomes.DNA linkage analysis is used to assist in the diagnosis of adult PCKD but the presence of multiple copies continues to complicate the development of reagents for direct genetic testing, at least of the 70% of PKD1 that is replicated elsewhere.

52) 1

The typical problem associated with sleep apnoea syndrome is excessive daytime somnolence. Associated with obesity, acromegaly, hypothyroidism and Cushing's syndrome. It is felt that the somnolence is due to the interruption of REM sleep by frequent episodes of waking due to apnoeic episodes.

53) 5

This is a typical case of Stevens-Johnson syndrome. Stevens-Johnson syndrome (SJS) is an immune-complex–mediated hypersensitivity complex that is a severe expression of erythema multiforme. It is now known also as erythema multiforme major. SJS typically involves the skin and the mucous membranes. While minor presentations may occur, significant involvement of oral, nasal, eye, vaginal, urethral, GI, and lower respiratory tract mucous membranes may develop in the course of the illness. GI and respiratory involvement may progress to necrosis.

SJS is a serious systemic disorder with the potential for severe morbidity and even death. The drugs most closely associated with causing Stevens-Johnson syndrome are antibacterials, sulfonamides, anticonvulsants, oxicam non-steroidal anti-inflammatory agents (piroxicam and tenoxicam), chlormezanone and allopurinol.

54) 2

This is typical of Clostridium infection with pseudomembranous colitis induced by prior treatment with broad spectrum antibiotics such as cefuroxime, augmentin and the macrolides. It is treated with oral vancomycin/metronidazole.

55) 4

Because the sympathetic plexus accompanying the internal carotid artery innervates sweat glands only to the medial forehead, facial anhydrosis does not occur significantly with postganglionic Horner syndrome

56) 1

This is a gastric MALT tumour. These are usually marginal zone B cell lymphomas and associated with an excellent prognosis. Low grade gastric MALT tumours associated with Helicobacter Pylori infection respond in over 80% to helicobacter eradication as the primary mode of treatment. Radiotherapy is considered but generally unnecessary.

57) 3

This patient has malignant hypertension with papiloedema, convulsions and pulmonary oedema (thus excluding the use of a beta-blocker in the acute setting). This constitutes a medical emergency with Nitroprusside being the treatment of choice.

58) 4

A blood sample needs to be drawn at least six hours after administration of digoxin to ensure adequate distribution.

59) 4

Lactulose, an osmotic diuretic causes hypomagnesaemia associated with diarrhoea, is not absorbed, does not affect the absorption of spironolactone and may be used in diabetics. It is used in patients with cirrhosis/hepatic encephalopathy to limit the proliferation of ammonia forming gut organisms and increase the clearance of protein load in the gut.

60) 3

NG feeds contain large amounts of salt and may be insufficient to match daily fluid losses. Hyperglycaemia would be unlikely unless the patient was already diabetic and was unable to manage the glucose load. The magnesium load in NG feeds would be insufficient to cause any clinical problem. Hypocalcaemia would be unlikely to present like this – tetany etc and should take more than five days to develop. Again there is no reason for hypophosphataemia.

61) 2

Odd question as all the agents can cause postural hypotension but one might think that the causative agent would be an alpha-blocker. However, the evidence suggests the contrary as amongst all the classes a beta-blocker is most associated with postural hypotension in the elderly. So, the correct answer is a beta-blocker yet it’s a lottery to determine if the RCP would have this down as their right answer. Other agents particularly associated with postural hypotension include the centrally acting agents like alpha Methyl Dopa and reserpine.

62) 3

The raised prolactin would most likely reflect stalk compression in this patient. Otherwise, the normal cortisol would be unhelpful as is the normal TSH. The elevated LH is a reflection of this patient being menopausal. GH concentrations are frequently undetectable as it is released episodically usually during the night.

63) 1

The diabetic male has evidence of liver disease with grossly elevated Ferritin suggesting a diagnosis of Haemochromatosis. A non-migratory polyarthritis would be suggestive although an oligoarthritis particularly of the hands and hip is more typical. Skin pigmentation rather than a rash is more typical. Myxoedema is not a feature of haemochromatosis. Gynaecomastia is a feature of liver disease/cirrhosis per se and not just haemochromatosis. However, chondrocalcinosis together with the chronic arthropathy is well recognized in association with haemochromatosis.

64) 1

Atenolol is a water soluble beta blocker, taken once daily and is not associated with drowsiness/sleep disturbance like the lipid-soluble beta-blockers. It is not associated with nausea or hesitancy of micturition and would be unlikely to produce significant postural hypotension in a hypertensive subject. However, fatigue is a frequent side effect which typically is felt two hours and beyond after taking the drug.

65) 4

The patient has a severe lithium overdose as reflected by markedly elevated Lithium concentrations and features of impaired consciousness, tremor and seizures. This needs urgent management. Activated charcoal does not bind Lithium effectively and is therefore ineffective except where co-ingestion of other poisons is suspected. Haemodialysis is the mainstay of treatment for acute Lithium toxicity.

66) 3

This is pseudomembranous colitis due to Clostridium Difficile secondary to Antibiotic usage for his COAD. Plain AXR is useful for diagnosing toxic dilatation but does not establish the diagnosis. Stool microscopy has no value but stool toxin assay is useful. A Patient with diarrhoea normally has involvement of the distal colon and rectum and sigmoidoscopy with biopsy is helpful for rapid diagnosis. Patients with involvement of right colon usually have little or no diarrhoea. Patient with diarrhoea normally has involvement of the distal colon and rectum and sigmoidoscopy with biopsy is helpful for rapid diagnosis. Patients with involvement of right colon usually have little or no diarrhoea.

67) 5

This diabetic has poor glycaemic control with renal impairment. With creatinine concentrations above 150 &micro;mol/l, metformin is not recommended due to the small risk of lactic acidosis. Therefore, the most appropriate treatment as recommended by NICE would be rosiglitazone as LFTs are normal and there is no suggestion of heart failure. Acarbose is poorly tolerated and is now rarely prescribed. Guar gum has little place in the treatment of diabetes. Repaglinide the non-sulphonylurea insulin secretagogue would have little benefit in conjunction with a traditional SU such as Gliclazide.

68) 3

Without being given too much here, this patient is a newly diagnosed diabetic as we are told he was previously fit and well. Therefore the most probable explanation for his blurred vision is osmotic changes.

69) 3

This patient has developed phenytoin toxicity which has been precipitated by cimetidine which inhibits cytochrome P450 metabolism of phenytoin. Phenytoin concentration is reduced by St John’s Wort, unaffected by amitriptyline which would however reduce seizure threshold; antacids may reduce phenytoin absorption and oestradiol metabolism may be increased by phenytoin.

70) 4

This patient may well have myeloma as reflected by the anaemia, leucopaenia and elevated non-albumin protein concentration. Thus plasma protein electrophoresis would be the investigation of choice in this patient.

71) 2

The history of non-itchy, circular, raised, smooth-surfaced lesions on the elbows, extensor aspects of the forearms and knuckles and the raised borders are suggestive of granuloma annulare. Discoid eczema tends to be scaly and pruritic in nature, psoriasis typically has a silvery scale and can be pruritic, urticaria lasts a few hours and is pruritic and tinea corporis is a fungal infection and is typically scaly and pruritic in nature. The most likely answer is therefore granuloma annulare. This can be associated with diabetes.

72) 4

This patient obviously has diabetes mellitus and the diagnosis should be confirmed with either a fasting plasma glucose above 7 mmol/l or a random plasma glucose above 11.1 mmol/l.

73) 3

The features of this blood film are anaemia, thrombocytosis, neutrophilia with roughly 55% neutrophils, 40% myelocytes with less than 5% blast cells. This is typical of Chronic Myeloid Leukaemia which usually has associated tender splenomegaly. Usually the Philadelphia chromosome is present in 95% of cases. Acute leukaemia is defined as blast cells constituting over 30% of cell type present. CML often ends in acute blastic transformation after a mean duration of approx 4 years.

74) 3

This is a really odd basic anatomy question which would be better suited to a surgical exam! However the only retroperitoneal structure is the pancreas, the body of which is in direct approximation to the anterior surface of the left kidney

75) 5

The Philadelphia chromosome (translocation 9;22) is present in approx 95% of subjects with CML. BCR-ABL gene fusion may be found in the 5% without Ph chromosome. Deletion of Ch13 is associated with a poorer prognosis in Multiple Myeloma.

76) 4

This patient is likely to have Addison's disease based upon her history, autoimmune disease, and presentation. She requires treatment with IV hydrocortisone which can be a life saving manoeuvre in acute hypoadrenalism.

77) 2

Although the SSRIs are also rarely associated with hyperprolactinaemia, the answer has to be Metoclopramide which is a dopamine antagonist that is typically associated with hyperprolactinaemia.

78) 1

The features of relief with alcohol, postural tremor and a slight and rather benign deterioration over five years suggests a diagnosis of Benign essential tremor. Physiological tremor usually affects the hands only. In this case there is titubation, again a feature of essential tremor. The mild features argue against Parkinson’s disease. The condition is typically slowly progressive and occurs from approximately 50 years.

79) 5

This patient has subclinical hypothyroidism as reflected by the normal T4 but elevated TSH. A hypercholesterolaemia with hypertriglyceridaemia is frequently associated due to impaired lipoprotein lipase function. The dyslipidaemia may well resolve following the appropriate replacement with thyroxine.

80) 4

The most likely explanation for the failure of an iron deficiency anaemia to respond to iron therapy in a menstruant female is poor compliance. It is likely that the dose that this patient is prescribed would be adequate and if not some response would still be expected. There is no evidence of a concomitant folate deficiency as suggested by the blood picture, which would also argue against Coeliac disease. Similarly, there is no evidence to suggest a sideroblastic anaemia where a raised MCV and increased ferritin may be expected.

81) 1

Unlike the other conditions, no one specific genetic defect has been identified to account for ankylosing spondylitis. Hungtindon’s chorea is autosomal dominant condition. Fragile X syndrome is due to a trinucleatide repeat at the FMR 1 gene on the X chromosome. Erythropoietic porphyria is an autosomal recessive condition as is Pendred’s syndrome.

82) 1

Unilateral occlusion (distal to Ant. Comm. origin) of Anterior Cerebral Artery produces contralateral sensorimotor deficits mainly involving the lower extremity with sparing of face and hands (think of the humunculus).

The Lateral Lenticulostriate artery is a branch of the middle cerebral artery. Occlusion causes damage to the internal capsule resulting in contralateral hemiparesis and sensory deficit. Speech may be affected (medial temporal lobe) as well as visual function (Meyer's loop: optic radiations affected).

Middle Cerebral Artery: Occlusion at the stem (proximal segment) results in:

• Contralateral hemiplegia affecting face, arm, and leg (lesser).

• Homonymous hemianopia - Ipsilateral head/eye deviation.

• If on left: global aphasia.

Posterior cerebral artery: A variety of neurological syndromes including:-

• Pure hemisensory loss

• visual field loss- a variety

• Visual agnosia

• Disorders of reading (alexia, dyslexia) and more..........

83) 5

The bisphosphonates of which alendronate is one, increase Bone mineralisation by inhibiting osteoclastic activity. They have been demonstrated in numerous studies to reduce subsequent risk of fracture.

84) 4

The patient is likely to have neurofibromatosis.

To be given the diagnosis of NF1, an individual must have at least two of the following features. Some people with NF1 have only two, while others can have several of these features:

• Six or more cafe-au-lait spots, or coffee-colored birthmarks, each measuring over an inch in adults (1/4 inch in children).

• Two of more neurofibromas or a plexiform neurofibroma.

• Freckles under the arm or in the groin region.

• A tumor of the nerve to the eye called an optic glioma.

• Two or more spots on the iris of the eye called Lisch nodules.

• A problem of one of the bones such as bowing of a leg, with or without a fracture.

• A parent, brother, sister, or child with NF1.

85) 4

This patient has a mild metabolic alkalosis with what appears to be respiratory compensation as reflected by the elevated pCO2. Amitriptyline overdose is associated with acidosis as is salicylate poisoning. Hepatic failure usually presents with acidosis. This type of picture is associated with prolonged vomiting (as in pregnancy), diarrhoea, diuretic therapy and in Cushing’s syndrome or in those receiving high dose corticosteroids. With no other information provided for this case, common things being common, one should select pregnancy as the best answer for a 20 year old female.

86) 5

There is a marked hypernatraemia with elevated chloride but normal potassium and urea in a patient with severe head injuries. The likely cause of this presentation is Diabetes Insipidus. Urine osmolality is therefore likely to be low. You cannot say that she has a hyperchloraemic acidosis as you do not have her bicarbonate concentration but if you assume that the anion gap would be normal i.e – 10 –12 then this would suggest that the bicarbonate is elevated suggesting either a metabolic alkalosis or respiratory acidosis with compensation. Although restoration of normal volaemia and osmolality is required giving 5% dextrose may exacerbate any cerebral oedema and so correction should be gradual.

87) 4

The most likely answer is gastric erosions based upon the fact that the incident has occured after only one dose of Aspirin without any prior history of chronic usage which is associated with oesophagitis and ulcer formation. Malignancy is unlikely because of history of being fit and also the age.

88) 1

This question is extremely poorly presented as no one, except for the RCP, measures total thyroid hormone concentrations. However, this patient has normal TSH, low total T4 with normal total T3 and really low free T4 which would suggest that she is taking T3 as replacement therapy. This may explain why no fT3 figures are provided. Consequently she is receiving adequate replacement as reflected by the normal TSH. She does not have sick euthyroidism as it states in the run in that these measurements were routine. Although TSH is normal and tT4 and fT4 low, secondary/tertiary hypothyroidism would not explain the plum normal total T3 concentration. She may well have had a thyroiditis such as Hashimoto’s to have given her the hypothyroidism originally but she is now on replacement therapy and the former would not explain her TFTs.

89) 4

This young woman has thromboembolic disease, the nephritic syndrome with positive anti-ds DNA antibodies suggests a diagnosis of SLE. Nephrotic syndrome in the absence of hypertension, active urinary sediment, or significant hypocomplementemia suggests membranous nephropathy. The thromboembolic disease may arise due to nephritic syndrome per se or from an associated antiphospholipid syndrome.

90) 5

The patient has a hepatitic picture in contrast to Cholestasis. Ferritin level is not too high to be considered for haemochromatosis and is an acute phase reactant being typically increased in any inflammatory process. NASH is very common and is typically encountered in Obese patients, presenting with a hepatitic picture with or without jaundice. Echo bright liver suggests fatty change in the liver seen in NASH.It was previously termed Idiopathic decompensated hepatitis and if not treated in terms of lowering BMI and reducing fat intake can lead onto irreversible cirrhosis. Gallstones are a distraction in this history.

91) 4

The presence of a small pericardial effusion on echo is quite common and in this patient who otherwise appears well, no further action is required.

92) 3

This one chap may be frustrated by his incapacity and would appear to having behavioural problems as a manifestation of his depression. Cannabis may actually relive these frustrations. There are no features to suggest an organic brain syndrome.

93) 1

Post-marketing surveillance/observational studies (phase IV studies) generally are designed to assess the potential side effects of new drugs but under everyday conditions and with a minimum of intervention. In contrast to the randomized controlled trials, PMS typically include patients from more extreme age groups, patients with comorbidity or other risk factors. In order to cover a wide spectrum of patients and to observe rare events with sufficiently high probability, PMS enroll a large number of patients, typically several thousands. Comparative efficacy has already been undertaken in Phase III studies (RCTs) but can also be undertaken as part of specific RCT studies later in the drugs development and potency usually in phase I and II studies.

94) 1

Amitriptyline has anticholinergic effects being associated with tacchycardia, dry mouth and urinary retention. This is not an effect of the SSRIs such as Venlafaxine and Fluoxetine. Diazepam, a benzodiazepine does not have anticholinergic effects. Zopiclone is a benzodiazepine like agent whose side effects include drowsiness.

95) 2

Eaton - Lambert Syndrome is characterised by proximal muscle weakness (the cranial nerves and respiratory muscles are usually spared), depressed or absent tendon reflexes and autonomic features (eg. dry mouth, impotence etc). 70% of cases are due to small cell lung cancer. Unlike myasthenia gravis exercise is associated with increasing muscle strength and there is a negative response to Tensilon. Electromyography is useful in confirming the diagnosis where repeated nerve stimulations cause a progressive increase in the size of the muscle action potential. Eaton (1905-1958) – U.S. neurologist at Mayo Clinic. Lambert (1915- ) – U.S neuro-physiologist at Mayo Clinic and Prof. of physiology at University of Minnesota.

96) 5

Not that much given away by this history just the issue of a prolonged bleed after prior dental extraction. The most likely diagnosis when considering this patient is von Willebrand’s disease which is an autosomal dominant condition and is one of the commonest bleeding disorders. Most cases are mild, with bleeding after only mild injury, particularly mucosal membrane injuries. The condition is due to a reduction or structural abnormality of von Willebrand's factor, which has the dual role of promoting normal platelet function and stabilising coagulation factor VIII. Von Willebrand's disease can give normal results on screening tests, and diagnosis may require specialist investigation. Most patients with mild disease respond to desmopressin (DDAVP), but clotting factor concentrates are needed for a minority.

97) 1

Amiodarone has been shown to be superior in maintaining SR following DC cardioversion of AF; however, it is associated with more toxic side effects than the other agents mentioned. Neither verapamil, diltiazem nor digoxin would be expected to maintain SR to any significant extent. Sotalol may be considered as a possible therapy but is less effective than amiodarone.

98) 4

The history suggests temporal arteritis irrespective of the history of anxiety and in view of the sight threatening nature of the disease, the patient should be commenced on steroids. Although the differential diagnosis is also trigeminal neuralgia steroids should be used here whilst awaiting diagnostic investigations as temporal arteritis may be sight threatening if left untreated.

99) 4

There are 40 true positive patients identified by Colonoscopy with colon cancer and 10 of these are false negatives as identified by CT. There are 360 patients without the disease with 20 identified as having cancer with CT (false positives).

100) 3

This is a classical presentation of Guillain-Barre with the gradual development of ascending weakness with autonomic involvement.

101) 5

Symptoms of GORD do not correlate with the mucosal appearances at endoscopy. Although endoscopy should be performed in cases that are not clear cut or do not respond to PPI, it is not mandatory. The risk with Barrett’s and hence the diagnosis of Barrett’s can only be clarified with biopsy.

Monitoring of pH is not a good guide to therapy but symptomatic improvement is a good guide to the efficacy of therapy. PPPIs can be given continuously where the diagnosis has been satisfactorily proven and relapse of symptoms persist after withdrawal.

102) 4

The most appropriate initial therapy for this patient who has a high risk of thrombo-embolic stroke is anticoagulation with warfarin maintaining an INR between 2-2.5. This should be the initial priority as he has already had one episode of TIA. The maintenance of sinus rhythm would be the next step and amiodarone or sotalol are options.

103) 2

This patient has a metabolic acidosis with an effort at respiratoty compensation as reflected by elevated pO2 and reduced pCO2. This could be due to poisoning or a condition such as Diabetic ketoacidosis. Vomiting would cause a metabolic alkalosis.

104) 4

This patient has anxiety symptoms due to withdrawal from alcohol. The typical symptoms include agitation, fever, sweats and tremor which are relived by alcohol. These symptoms usually peak after about 72 hours and may last a week or more but should have improved after three weeks.

105) 5

This patient has hypercalcaemia with a lowish phosphate concentration but an inappropriately normal PTH concentration suggesting hyper ## no greeklish please! ## - ## no greeklish please! ## - ## no greeklish please! ## - ## no greeklish please! ## - ## no greeklish please&#33; ## - oxi fragolevantika grapste kalytera sta agglika - fragolevantika grapste kalytera ## no greeklish please&#33; ## - oxi fragolevantika grapste kalytera sta agglika - agglika - fragolevantika grapste kalytera ## no greeklish please! ## - ## no greeklish please! ## - ## no greeklish please&#33; ## - oxi fragolevantika grapste kalytera sta agglika - fragolevantika grapste kalytera ## no greeklish please&#33; ## - oxi fragolevantika grapste kalytera sta agglika - agglika - fragolevantika grapste kalytera ## no greeklish please&#33; ## - oxi fragolevantika grapste kalytera sta agglika - agglika - agglika - fragolevantika grapste kalytera ## no greeklish please! ## - ## no greeklish please! ## - ## no greeklish please! ## - ## no greeklish please&#33; ## - oxi fragolevantika grapste kalytera sta agglika - fragolevantika grapste kalytera ## no greeklish please&#33; ## - oxi fragolevantika grapste kalytera sta agglika - agglika - fragolevantika grapste kalytera ## no greeklish please! ## - ## no greeklish please! ## - ## no greeklish please&#33; ## - oxi fragolevantika grapste kalytera sta agglika - fragolevantika grapste kalytera ## no greeklish please&#33; ## - oxi fragolevantika grapste kalytera sta agglika - agglika - fragolevantika grapste kalytera ## no greeklish please&#33; ## - oxi fragolevantika grapste kalytera sta agglika - agglika - agglika - fragolevantika grapste kalytera ## no greeklish please! ## - ## no greeklish please! ## - ## no greeklish please&#33; ## - oxi fragolevantika grapste kalytera sta agglika - fragolevantika grapste kalytera ## no greeklish please&#33; ## - oxi fragolevantika grapste kalytera sta agglika - agglika - fragolevantika grapste kalytera ## no greeklish please&#33; ## - oxi fragolevantika grapste kalytera sta agglika - agglika - agglika - agglika - fragolevantika grapste kalytera ## no greeklish please! ## - ## no greeklish please! ## - ## no greeklish please! ## - ## no greeklish please! ## - ## no greeklish please&#33; ## - oxi fragolevantika grapste kalytera sta agglika - fragolevantika grapste kalytera ## no greeklish please&#33; ## - oxi fragolevantika grapste kalytera sta agglika - agglika - fragolevantika grapste kalytera ## no greeklish please! ## - ## no greeklish please! ## - ## no greeklish please&#33; ## - oxi fragolevantika grapste kalytera sta agglika - fragolevantika grapste kalytera ## no greeklish please&#33; ## - oxi fragolevantika grapste kalytera sta agglika - agglika - fragolevantika grapste kalytera ## no greeklish please&#33; ## - oxi fragolevantika grapste kalytera sta agglika - agglika - agglika - fragolevantika grapste kalytera ## no greeklish please! ## - ## no greeklish please! ## - ## no greeklish please! ## - ## no greeklish please&#33; ## - oxi fragolevantika grapste kalytera sta agglika - fragolevantika grapste kalytera ## no greeklish please&#33; ## - oxi fragolevantika grapste kalytera sta agglika - agglika - fragolevantika grapste kalytera ## no greeklish please! ## - ## no greeklish please! ## - ## no greeklish please&#33; ## - oxi fragolevantika grapste kalytera sta agglika - fragolevantika grapste kalytera ## no greeklish please&#33; ## - oxi fragolevantika grapste kalytera sta agglika - agglika - fragolevantika grapste kalytera ## no greeklish please&#33; ## - oxi fragolevantika grapste kalytera sta agglika - agglika - agglika - fragolevantika grapste kalytera ## no greeklish please! ## - ## no greeklish please! ## - ## no greeklish please&#33; ## - oxi fragolevantika grapste kalytera sta agglika - fragolevantika grapste kalytera ## no greeklish please&#33; ## - oxi fragolevantika grapste kalytera sta agglika - agglika - fragolevantika grapste kalytera ## no greeklish please&#33; ## - oxi fragolevantika grapste kalytera sta agglika - agglika - agglika - agglika - fragolevantika grapste kalytera ## no greeklish please! ## - ## no greeklish please! ## - ## no greeklish please! ## - ## no greeklish please&#33; ## - oxi fragolevantika grapste kalytera sta agglika - fragolevantika grapste kalytera ## no greeklish please&#33; ## - oxi fragolevantika grapste kalytera sta agglika - agglika - fragolevantika grapste kalytera ## no greeklish please! ## - ## no greeklish please! ## - ## no greeklish please&#33; ## - oxi fragolevantika grapste kalytera sta agglika - fragolevantika grapste kalytera ## no greeklish please&#33; ## - oxi fragolevantika grapste kalytera sta agglika - agglika - fragolevantika grapste kalytera ## no greeklish please&#33; ## - oxi fragolevantika grapste kalytera sta agglika - agglika - agglika - fragolevantika grapste kalytera ## no greeklish please! ## - ## no greeklish please! ## - ## no greeklish please&#33; ## - oxi fragolevantika grapste kalytera sta agglika - fragolevantika grapste kalytera ## no greeklish please&#33; ## - oxi fragolevantika grapste kalytera sta agglika - agglika - fragolevantika grapste kalytera ## no greeklish please&#33; ## - oxi fragolevantika grapste kalytera sta agglika - agglika - agglika - agglika - agglika -oidism which is a relatively common disorder amongst elderly females. The story of the breast lump in this case is endeavouring to throw the candidate. Vitamin D excess would be expected to cause an elevated Phosphate.

106) 1

Cerebellar hemorrhage:The most common symptoms are of severe nausea and vomiting and ataxia. Headache may be severe. Patients with cerebellar hemorrhage can rapidly become comatose within hours after the onset from herniation, because of its limited space in the posterior fossa

Pontine hemorrhage: There are numerous nuclei located within the pons. Rapidly deteriorating level of consiousness, impaired extraocular movement and extensive sensorimotor deficits are clinical clues to pontine hemorrhage.

Basal Ganglia haemorrage: Contralateral hemiparesis, hemisensory loss, or hemi-inattention. Aphasia, especially nonfluency and impaired comprehension, is frequently seen if haemorrhage occurs in the posterior limb of the left internal capsule.

107) 3

This patient developed anaphylaxis during a surgical procedure and it appears that she had problems with allergies during dental treatment and whilst assisting during sterile procedures. This would suggest that she is allergic to latex rather than induction agents or local anaesthesia as latex would be present in all three of the above procedures. Systemic mastocytosis is a disease which usually affects the elderly and is associated with urticaria pigmentosa, diarrhoea, hypotension, sclerotic bone changes and mast cell infiltration of organs such as spleen, liver kidneys.

108) 4

Weakness of wrist extension with wrist drop and weakness of finger extension are typical of a radial nerve lesion.

109) 1

This patient has desaturation with patchy infiltration on CXR suggesting a diagnosis of amiodarone-induced lung disease. Usually the presentation is insidious and the disorder associated with the cumulative dose. Treatment depends on withdrawing amiodarone and steroids. Differential diagnosis is any lymphangitis/pnemonitis but High resolution CT can help by demonstration of radio-dense plaques etc.

110) 1

Inspection of the maximal expiratory and inspiratory flow-volume curve is currently the simplest method to establish the presence of upper airway obstruction associated with a retrosternal goitre. This may be present in up to 40% of patients with retrosternal goitre and generally requires at least 50% obstruction of the airway before symptoms arise.

111) 3

This patient is diabetic and has proteinuria. Although diabetic nephropathy usually takes 5 or more years to evolve, this patient is likely to have had the condition for many years prior to it now being diagnosed. Ischaemic nephropathy, due to renal artery stenosis is unlikely in the presence of a normal renal ultrasound. Analgesic nephropathy would be a consequence of NSAIDs not paracetamol. Hypertensive retinopathy is a possibility but is less likely in the context of her proteinuria and moderately elevated BP.

112) 5

Partial opoid agonists when used in association with morphine may produce a reduction in it's analgesic effect due to partial antagonism.

113) 5

Urinary incontinence would be particularly unusual in association with a depression and would be far more typical of dementia/normal pressure hydrocephalus. The other symptoms may be found in dementia particularly impaired memory and concern over memory deficits.

114) 3

Aortic stenosis is associated with a worse prognosis when accompanied by left ventricular dysfunction. Other predictors of a poorer prognosis include increasing gradient across the valve (above 70 mmHg), Age of patient and symptomatology. Although the severity of valvular calcification is prognostically important in an asymptomatic patient the most important predictor is LV function.

115) 3

The best agent for acute agitation in this elderly patient would be Haloperidol probably administered IM. Diazepam is too short acting to be administered to this patient and may precipitate respiratory depression whilst Temazepam oral has too long a duration of action to be of any use as an acute sedative in the elderly. Trazadone an anti-depressant is useful orally as a chronic anxiolytic. Other useful agents would be Lorazepam IM or Respiridone.

116) 2

The proportion of ‘true negatives’ not having had a MI correctly identified by this test is called the Negative predictive value and refers to the number accurately identified to not have MI by the new test over the number without MI identified by the test + those wrongly identified as not having had an MI. specifcity is the number without MI accurately predicted. Sensitivity refers to the number correctly identified with MI by the new test. A positive predictive value refers to the number accurately identified with MI by the test over the number accurately identified with MI + those wrongly identified with MI.

117) 3

Anticoagulation with subcutaneous heparin is recommended in most guidelines with LMWH being a suitable alternative. The latter has the advantage of requiring no monitoring but is a less well established therapy in pregnancy.

She cannot be treated with IV heparin throughout her pregnancy and Warfarin is also contra-indicated due to teratogenicity. Aspirin provides no demonstrable prophylactic value for venous thromboembolism.

118) 4

The specificity of a test is the probability that a test will produce a true negative result when used on an unaffected population, whereas the sensitivity of a test is the probability that it will produce a true positive result when used on an affected population (as determined by a reference or "gold standard"). The positive predictive value of a test is the probability that a person is affected when a positive test result is observed. The negative predictive value of a test is the probability that a person is not affected when a negative test result is observed. Accuracy is expressed through the above four parameters.

119) 3

The patient is homozygous for CF (CF/CF) and the father is heterozygous for CF (CF/N). Thus there is a 50% chance that her child will be homozygous for CF and a 50% chance that the child will be a carrier.

120) 4

In this middle aged female, the acute arthritis of shoulders and wrists together with synovitis and raised ESR are highly suggestive of acute Rhematoid Arthritis. Weakness and myalgia would be expected with polymyositis and a rash would be expected with SLE with little evidence of a synovitis. There is no prior precipitant to suggest a reactive arthritis and synovitis would be again unusual. PMR would be less likely in this age group and weakness, weight loss without synovitis would be expected.

121) 4

All her parameters are normal except calcium which is at the lower limit of normal. Calcium in the gut normally gets absorbed in the large bowel and laxative abuse causes lower bowel hurry causing decreased absorption of calcium. Lactose intolerance is unlikely because of the duration and age. Any form of colitis and other malabsorpion syndromes like ileocaecal tuberculosis causes raised CRP and decreased albumin with other blood abnormalities such as anaemia.

122) 5

Elevated troponin concentrations are highly specific for cardiac injury/infarction and are usually normal in patients with ischaemic heart disease/angina but may also be elevated in patients with pulmonary embolism or renal failure (reduced excretion). Concentrations typically peak 12-24 hrs after myocardial injury and remain elevated for 7-14 days. Studies reveal that raised troponins are far more specific than CK-MB.

123) 2

Ectopia lentis/ subluxation of the lens is associated with Ehlers Danlos syndrome, Marfan’s syndrome, Weil-Marchesani syndrome (short stature, skeletal abnormalities and ectopia lentis), Refsum’s disease but in this case homocystinuria is likely due to the associated mental retardation. Patients typically have fair skin with coarse hair, osteoporosis, mental retardation (nearly 50%), seizure disorder, marfanoid habitus, and and increased thromboembolic risk. Metachromatic leukodystrophy is a lysosomal storage disorder characterised by lipid (sulfatide) accumulation in the CNS and associated with psychomotor retardation in infants with optic atrophy and blindness.

124) 3

The diagnosis is likely to be atherosclerotic renal artery stenosis as suggested by the asymmetric reduction in renal size, with mild proteinuria quite common in the condition. Investigations include Captopril Renogarphy, MR angiography which is virtually as good as renal arteriography. None of the other investigations are appropriate for RAS.

Link to comment
Μοιράσου σε άλλους δικτυακούς τόπους

  • 7 months later...
Γεια!

Ψάχνω βιβλία για προετοιμασία για MRCP.Γνωρίζεις κάποια να μου προτείνεις?

Ευχαριστώ.

Υποθέτω πως εννοείς το MRCP 1.

Οπότε :

- "Essential revision notes for MRCP" του Philip Kalra.

Είναι ένα πολύ καλό βιβλίο που όπως το λέει ο τίτλος είναι "notes", δηλ. έχει πολύ λίγες εικόνες και δεν εξηγεί παθογένειες -εκτός από λίγες περιπτώσεις-, συγκεντρώνει όμως πολλά χρησιμα στοιχεία από όλα τα απαραίτητα κεφάλαια και τα παρουσιάζει με φιλικό για τον αναγνώστη τρόπο. Δεν πάει κανείς να δώσει χωρίς αυτό.

- "Clinical Medicine", των Kumar and Clarke. Είναι πολύ καλό για να ανατρέχεις όταν το πρώτο δεν σε καλύπτει. Και γενικά πολύ καλό για να υπάρχει στη βιβλιοθήκη σου.

- Πολλές μα πάρα πολλές ερωτήσεις για να δεις το στυλ και να εξοικειωθείς με αυτο.

http://www.pastest.co.uk/public/subjectPro....aspx?subject=1

Πληρώνεις συνδρομή για τις εβδομάδες ή τους μήνες που θες και έχεις πρόσβαση στη βάση δεδομένων με τις ερωτήσεις. Οι ερωτήσεις των εξετάσεων δεν είναι βέβαια από δεξαμενή, αλλά αν έχεις κατανοήσει πώς δουλεύουν θα ξέρεις πώς να διαβάσεις και τι να επιλέξεις όταν έρθει η ώρα να δώσεις.

Προσωπική μου άποψη ότι τα courses ειναι περιττά.

Θέλει υπομονή και αρκετό-πολύ διάβασμα.

Καλή σου επιτυχία! :):)

Link to comment
Μοιράσου σε άλλους δικτυακούς τόπους

Archived

This topic is now archived and is closed to further replies.

×
×
  • Δημιουργία νέας...

Important Information

By using this site, you agree to our Terms of Use.